Lesions on the hands, high aminotransferase levels

Article Type
Changed
Mon, 01/08/2018 - 14:39
Display Headline
Lesions on the hands, high aminotransferase levels

Figure 1. Crusted erosions, milia, scars, and considerable hair growth on sun-exposed surfaces of both hands.
A 64-year-old man presents with a 10-day history of painful vesicles and erosions on the dorsa of the hands that appeared after sun exposure (Figure 1). He reports that for the past year he has noticed hyperpigmentation and periorbital hypertrichosis (Figure 2). He has a history of alcohol abuse and chronic hepatitis B and C infection, and he smokes twenty cigarettes (one pack) per week. He is not taking any medications.

Figure 2. Modest hypertrichosis lateral to the eyebrows and several crusted erosions on the nose and forehead are consistent with the diagnosis of porphyria cutanea tarda.
Routine blood tests show mild elevation of aminotransferase levels but no other abnormalities.

Q: Which is the most likely diagnosis?

  • Addison disease
  • Lupus erythematosus
  • Polymorphous light eruption
  • Porphyria cutanea tarda
  • Bullous pemphigoid


A: Urine testing, including examination under ultraviolet light with a Wood lamp, indicates porphyria cutanea tarda. This is the most common porphyria, occurring mainly in men. Its true prevalence is not known but is estimated to be from 1:5,000 to 1:25,000.1

There are three types of porphyria cutanea tarda. About 80% of cases are type I, also referred to as “sporadic.” In type I, levels of uroporphyrinogen decarboxylase (UROD) in red blood cells are normal, but are low in the liver during episodes of the disease. In type II, UROD levels are about 50% below normal in all tissues. Type III is similar to type I, except that it occurs in more than one family member.

The genetic mutation that produces a deficiency of UROD leads to an excess of uroporphyrins and porphyrins that are partially decarboxylated and that irreversibly oxidize. When they are deposited in the skin and the skin is exposed to the sun, they cause the classic cutaneous manifestations.1

Risk factors2 for porphyria cutanea tarda can be extrinsic (eg, high iron blood levels,2,3 excessive ethanol intake, hepatitis C,2,4 human immunodeficiency virus, estrogen use,5 dialysis for end-stage renal disease) or intrinsic (altered iron metabolism or cytochrome P450 function2).

CLINICAL PRESENTATION

The cardinal symptom of porphyria cutanea tarda is photosensitivity, with the development of chronic blistering lesions on sun-exposed areas such as the hands, face, and forearms. Fluid-filled vesicles develop and rupture easily, and the denuded areas become crusted and heal slowly.5 Secondary infections can occur. Previous areas of blisters may appear atrophic, brownish, or violaceous. Small white plaques (milia) are also common and may precede or follow vesicle formation. These cutaneous lesions, however, are not specific to porphyria cutanea tarda and can appear in variegated porphyria and coproporphyria. Hypertrichosis5 and hyperpigmentation are usually present, mainly over the cheekbones and around the eyes. Patches of alopecia and hypopigmented sclerodermiform lesions may also be observed.

Porphyria cutanea tarda is usually accompanied by alterations in liver metabolism, affecting mainly aminotransferases and gammaglutamyltransferase. The absence of hepatitis C infection does not rule out porphyria cutanea tarda. About 50% of patients have pathologic structural changes in the liver such as lobular necrosis or fibrotic tracts, and 15% of patients have cirrhosis at presentation.6 The risk of hepatocellular carcinoma is clearly increased.6 Hepatitis C virus infection, iron overload, and excessive ethanol intake lead to a more severe liver disease.

DIAGNOSIS

The diagnosis of porphyria cutanea tarda is strongly suggested by the characteristic skin lesions in sun-exposed areas, but confirmation requires demonstration of high levels of uroporphyrins or coproporphyrins, or both.

Porphyrins accumulate in the liver, plasma, urine, and feces. Plasma porphyrin levels in porphyria cutanea tarda are usually above 10 μg/dL (normal < 1.4 μg/dL), and plasma fluorescence scanning usually shows a maximum fluorescence emission at an excitation wavelength of 619 nm. In this patient, however, the definitive diagnosis was made by chromatographic separation and the quantification of porphyrins in the urine and feces, which showed a predominance of uroporphyrins and heptacarboxyporphyrins in the urine and an excess of isocoproporphyrins in the feces.1

Analysis of UROD activity in erythrocytes can help determine the type of porphyria cutanea tarda. Type I and type III and porphyria cutanea tarda secondary to hepatotoxin exposure have normal levels, whereas type II and the hepatoerythropoietic form have abnormally low levels. Examination of the urine with a Wood lamp reveals coral pink fluorescence due to elimination of porphyrins, and this is another diagnostic clue.

Conditions that need to be ruled out include viral infection with hepatitis B or C or human immunodeficiency virus, iron overload, and hereditary hemochromatosis. Serum alpha fetoprotein level assessment, liver ultrasonography, or even biopsy may be indicated to exclude hepatocellular carcinoma.

TREATMENT

Once secondary causes of porphyria are excluded or treated (eg, advising the patient to avoid alcohol, discontinuing estrogens or iron intake), the next step in management is to reduce the patient’s porphyrin and iron loads. Phlebotomy is the standard way to reduce stores of iron throughout the body and particularly in the liver. It works by interrupting iron-mediated oxidative inhibition of hepatic UROD and the oxidation of hepatic porphyrinogens to porphyrinogens.

This adjustment must be gradual, with about 450 mL of blood removed at intervals of 1 to 2 weeks.7 This improves the cutaneous symptoms progressively, with resolution of vesicles in 2 to 3 months, improvement of skin fragility in 6 to 9 months, and normalization of porphyrin levels in 13 months. The scleroderma, atrophy, hyperpigmentation, and hypertrichosis respond more slowly and may take years to resolve.

Porphyria cutanea tarda can recur, usually with new exposure to risk factors. Treatment by phlebotomy may be stopped when the serum ferritin level has reached low-normal levels; the porphyrin levels may not yet be normal at that point but may continue to decline without additional phlebotomy sessions.

If phlebotomy is contraindicated, alternatives include iron chelation with deferoxamine (Desferal),7 or a low dose of chloroquine (Aralen) (125–250 mg orally twice a week) or hydroxychloroquine (Plaquenil) (100 mg orally twice a week) to avoid acute hepatic damage that may be caused by the release of large amounts of porphyrins that accompany standard dosing levels.

References
  1. Elder GH. Porphyria cutanea tarda. Semin Liver Dis 1998; 18:6775.
  2. Mendez M, Rossetti MV, Del C, Batlle AM, Parera VE. The role of inherited and acquired factors in the development of porphyria cutanea tarda in the Argentinean population. J Am Acad Dermatol 2005; 52:417424.
  3. Bonkovsky HL, Poh-Fitzpatrick M, Pimstone N, et al. Porphyria cutanea tarda, hepatitis C, and HFE gene mutations in North America. Hepatology 1998; 27:16611669.
  4. Ali A, Zein NN. Hepatitis C infection: a systemic disease with extrahepatic manifestations. Cleve Clin J Med 2005; 72:10051016.
  5. Grossman ME, Poh-Fitzpatrick MB. Porphyria cutanea tarda. diagnosis and management. Med Clin North Am 1980; 64:807827.
  6. Cortés JM, Oliva H, Paradinas FJ, Hernandez-Guío C. The pathology of the liver in porphyria cutanea tarda. Histopathology 1980; 4:471485.
  7. Rocchi E, Gibertini P, Cassanelli M, et al. Iron removal therapy in porphyria cutanea tarda: phlebotomy versus slow subcutaneous desferrioxamine infusion. Br J Dermatol 1986; 114:621629.
Article PDF
Author and Disclosure Information

Manuel Gil-Mosquera, MD
Instituto Madrileño de la Salud (IMSALUD), Madrid, Spain

Sergio Vañó-Galván, MD
Department of Dermatology, Ramón y Cajal University Hospital, Madrid, Spain

Ruth Gomez-Guerra, MD
Department of Dermatology, Ramón y Cajal University Hospital, Madrid, Spain

Pedro Jaén, MD
Chief, Department of Dermatology, Ramón y Cajal University Hospital, Madrid, Spain

Marina Gil-Mosquera, MD
San Carlos University Hospital, Madrid, Spain

Address: Manuel Gil-Mosquera, MD, Department of Dermatology, Ramón y Cajal University Hospital, Carretera Colmenar km 9100, 28034 Madrid, Spain; e-mail vesalio30@hotmail.com

Issue
Cleveland Clinic Journal of Medicine - 77(1)
Publications
Topics
Page Number
34-36
Sections
Author and Disclosure Information

Manuel Gil-Mosquera, MD
Instituto Madrileño de la Salud (IMSALUD), Madrid, Spain

Sergio Vañó-Galván, MD
Department of Dermatology, Ramón y Cajal University Hospital, Madrid, Spain

Ruth Gomez-Guerra, MD
Department of Dermatology, Ramón y Cajal University Hospital, Madrid, Spain

Pedro Jaén, MD
Chief, Department of Dermatology, Ramón y Cajal University Hospital, Madrid, Spain

Marina Gil-Mosquera, MD
San Carlos University Hospital, Madrid, Spain

Address: Manuel Gil-Mosquera, MD, Department of Dermatology, Ramón y Cajal University Hospital, Carretera Colmenar km 9100, 28034 Madrid, Spain; e-mail vesalio30@hotmail.com

Author and Disclosure Information

Manuel Gil-Mosquera, MD
Instituto Madrileño de la Salud (IMSALUD), Madrid, Spain

Sergio Vañó-Galván, MD
Department of Dermatology, Ramón y Cajal University Hospital, Madrid, Spain

Ruth Gomez-Guerra, MD
Department of Dermatology, Ramón y Cajal University Hospital, Madrid, Spain

Pedro Jaén, MD
Chief, Department of Dermatology, Ramón y Cajal University Hospital, Madrid, Spain

Marina Gil-Mosquera, MD
San Carlos University Hospital, Madrid, Spain

Address: Manuel Gil-Mosquera, MD, Department of Dermatology, Ramón y Cajal University Hospital, Carretera Colmenar km 9100, 28034 Madrid, Spain; e-mail vesalio30@hotmail.com

Article PDF
Article PDF

Figure 1. Crusted erosions, milia, scars, and considerable hair growth on sun-exposed surfaces of both hands.
A 64-year-old man presents with a 10-day history of painful vesicles and erosions on the dorsa of the hands that appeared after sun exposure (Figure 1). He reports that for the past year he has noticed hyperpigmentation and periorbital hypertrichosis (Figure 2). He has a history of alcohol abuse and chronic hepatitis B and C infection, and he smokes twenty cigarettes (one pack) per week. He is not taking any medications.

Figure 2. Modest hypertrichosis lateral to the eyebrows and several crusted erosions on the nose and forehead are consistent with the diagnosis of porphyria cutanea tarda.
Routine blood tests show mild elevation of aminotransferase levels but no other abnormalities.

Q: Which is the most likely diagnosis?

  • Addison disease
  • Lupus erythematosus
  • Polymorphous light eruption
  • Porphyria cutanea tarda
  • Bullous pemphigoid


A: Urine testing, including examination under ultraviolet light with a Wood lamp, indicates porphyria cutanea tarda. This is the most common porphyria, occurring mainly in men. Its true prevalence is not known but is estimated to be from 1:5,000 to 1:25,000.1

There are three types of porphyria cutanea tarda. About 80% of cases are type I, also referred to as “sporadic.” In type I, levels of uroporphyrinogen decarboxylase (UROD) in red blood cells are normal, but are low in the liver during episodes of the disease. In type II, UROD levels are about 50% below normal in all tissues. Type III is similar to type I, except that it occurs in more than one family member.

The genetic mutation that produces a deficiency of UROD leads to an excess of uroporphyrins and porphyrins that are partially decarboxylated and that irreversibly oxidize. When they are deposited in the skin and the skin is exposed to the sun, they cause the classic cutaneous manifestations.1

Risk factors2 for porphyria cutanea tarda can be extrinsic (eg, high iron blood levels,2,3 excessive ethanol intake, hepatitis C,2,4 human immunodeficiency virus, estrogen use,5 dialysis for end-stage renal disease) or intrinsic (altered iron metabolism or cytochrome P450 function2).

CLINICAL PRESENTATION

The cardinal symptom of porphyria cutanea tarda is photosensitivity, with the development of chronic blistering lesions on sun-exposed areas such as the hands, face, and forearms. Fluid-filled vesicles develop and rupture easily, and the denuded areas become crusted and heal slowly.5 Secondary infections can occur. Previous areas of blisters may appear atrophic, brownish, or violaceous. Small white plaques (milia) are also common and may precede or follow vesicle formation. These cutaneous lesions, however, are not specific to porphyria cutanea tarda and can appear in variegated porphyria and coproporphyria. Hypertrichosis5 and hyperpigmentation are usually present, mainly over the cheekbones and around the eyes. Patches of alopecia and hypopigmented sclerodermiform lesions may also be observed.

Porphyria cutanea tarda is usually accompanied by alterations in liver metabolism, affecting mainly aminotransferases and gammaglutamyltransferase. The absence of hepatitis C infection does not rule out porphyria cutanea tarda. About 50% of patients have pathologic structural changes in the liver such as lobular necrosis or fibrotic tracts, and 15% of patients have cirrhosis at presentation.6 The risk of hepatocellular carcinoma is clearly increased.6 Hepatitis C virus infection, iron overload, and excessive ethanol intake lead to a more severe liver disease.

DIAGNOSIS

The diagnosis of porphyria cutanea tarda is strongly suggested by the characteristic skin lesions in sun-exposed areas, but confirmation requires demonstration of high levels of uroporphyrins or coproporphyrins, or both.

Porphyrins accumulate in the liver, plasma, urine, and feces. Plasma porphyrin levels in porphyria cutanea tarda are usually above 10 μg/dL (normal < 1.4 μg/dL), and plasma fluorescence scanning usually shows a maximum fluorescence emission at an excitation wavelength of 619 nm. In this patient, however, the definitive diagnosis was made by chromatographic separation and the quantification of porphyrins in the urine and feces, which showed a predominance of uroporphyrins and heptacarboxyporphyrins in the urine and an excess of isocoproporphyrins in the feces.1

Analysis of UROD activity in erythrocytes can help determine the type of porphyria cutanea tarda. Type I and type III and porphyria cutanea tarda secondary to hepatotoxin exposure have normal levels, whereas type II and the hepatoerythropoietic form have abnormally low levels. Examination of the urine with a Wood lamp reveals coral pink fluorescence due to elimination of porphyrins, and this is another diagnostic clue.

Conditions that need to be ruled out include viral infection with hepatitis B or C or human immunodeficiency virus, iron overload, and hereditary hemochromatosis. Serum alpha fetoprotein level assessment, liver ultrasonography, or even biopsy may be indicated to exclude hepatocellular carcinoma.

TREATMENT

Once secondary causes of porphyria are excluded or treated (eg, advising the patient to avoid alcohol, discontinuing estrogens or iron intake), the next step in management is to reduce the patient’s porphyrin and iron loads. Phlebotomy is the standard way to reduce stores of iron throughout the body and particularly in the liver. It works by interrupting iron-mediated oxidative inhibition of hepatic UROD and the oxidation of hepatic porphyrinogens to porphyrinogens.

This adjustment must be gradual, with about 450 mL of blood removed at intervals of 1 to 2 weeks.7 This improves the cutaneous symptoms progressively, with resolution of vesicles in 2 to 3 months, improvement of skin fragility in 6 to 9 months, and normalization of porphyrin levels in 13 months. The scleroderma, atrophy, hyperpigmentation, and hypertrichosis respond more slowly and may take years to resolve.

Porphyria cutanea tarda can recur, usually with new exposure to risk factors. Treatment by phlebotomy may be stopped when the serum ferritin level has reached low-normal levels; the porphyrin levels may not yet be normal at that point but may continue to decline without additional phlebotomy sessions.

If phlebotomy is contraindicated, alternatives include iron chelation with deferoxamine (Desferal),7 or a low dose of chloroquine (Aralen) (125–250 mg orally twice a week) or hydroxychloroquine (Plaquenil) (100 mg orally twice a week) to avoid acute hepatic damage that may be caused by the release of large amounts of porphyrins that accompany standard dosing levels.

Figure 1. Crusted erosions, milia, scars, and considerable hair growth on sun-exposed surfaces of both hands.
A 64-year-old man presents with a 10-day history of painful vesicles and erosions on the dorsa of the hands that appeared after sun exposure (Figure 1). He reports that for the past year he has noticed hyperpigmentation and periorbital hypertrichosis (Figure 2). He has a history of alcohol abuse and chronic hepatitis B and C infection, and he smokes twenty cigarettes (one pack) per week. He is not taking any medications.

Figure 2. Modest hypertrichosis lateral to the eyebrows and several crusted erosions on the nose and forehead are consistent with the diagnosis of porphyria cutanea tarda.
Routine blood tests show mild elevation of aminotransferase levels but no other abnormalities.

Q: Which is the most likely diagnosis?

  • Addison disease
  • Lupus erythematosus
  • Polymorphous light eruption
  • Porphyria cutanea tarda
  • Bullous pemphigoid


A: Urine testing, including examination under ultraviolet light with a Wood lamp, indicates porphyria cutanea tarda. This is the most common porphyria, occurring mainly in men. Its true prevalence is not known but is estimated to be from 1:5,000 to 1:25,000.1

There are three types of porphyria cutanea tarda. About 80% of cases are type I, also referred to as “sporadic.” In type I, levels of uroporphyrinogen decarboxylase (UROD) in red blood cells are normal, but are low in the liver during episodes of the disease. In type II, UROD levels are about 50% below normal in all tissues. Type III is similar to type I, except that it occurs in more than one family member.

The genetic mutation that produces a deficiency of UROD leads to an excess of uroporphyrins and porphyrins that are partially decarboxylated and that irreversibly oxidize. When they are deposited in the skin and the skin is exposed to the sun, they cause the classic cutaneous manifestations.1

Risk factors2 for porphyria cutanea tarda can be extrinsic (eg, high iron blood levels,2,3 excessive ethanol intake, hepatitis C,2,4 human immunodeficiency virus, estrogen use,5 dialysis for end-stage renal disease) or intrinsic (altered iron metabolism or cytochrome P450 function2).

CLINICAL PRESENTATION

The cardinal symptom of porphyria cutanea tarda is photosensitivity, with the development of chronic blistering lesions on sun-exposed areas such as the hands, face, and forearms. Fluid-filled vesicles develop and rupture easily, and the denuded areas become crusted and heal slowly.5 Secondary infections can occur. Previous areas of blisters may appear atrophic, brownish, or violaceous. Small white plaques (milia) are also common and may precede or follow vesicle formation. These cutaneous lesions, however, are not specific to porphyria cutanea tarda and can appear in variegated porphyria and coproporphyria. Hypertrichosis5 and hyperpigmentation are usually present, mainly over the cheekbones and around the eyes. Patches of alopecia and hypopigmented sclerodermiform lesions may also be observed.

Porphyria cutanea tarda is usually accompanied by alterations in liver metabolism, affecting mainly aminotransferases and gammaglutamyltransferase. The absence of hepatitis C infection does not rule out porphyria cutanea tarda. About 50% of patients have pathologic structural changes in the liver such as lobular necrosis or fibrotic tracts, and 15% of patients have cirrhosis at presentation.6 The risk of hepatocellular carcinoma is clearly increased.6 Hepatitis C virus infection, iron overload, and excessive ethanol intake lead to a more severe liver disease.

DIAGNOSIS

The diagnosis of porphyria cutanea tarda is strongly suggested by the characteristic skin lesions in sun-exposed areas, but confirmation requires demonstration of high levels of uroporphyrins or coproporphyrins, or both.

Porphyrins accumulate in the liver, plasma, urine, and feces. Plasma porphyrin levels in porphyria cutanea tarda are usually above 10 μg/dL (normal < 1.4 μg/dL), and plasma fluorescence scanning usually shows a maximum fluorescence emission at an excitation wavelength of 619 nm. In this patient, however, the definitive diagnosis was made by chromatographic separation and the quantification of porphyrins in the urine and feces, which showed a predominance of uroporphyrins and heptacarboxyporphyrins in the urine and an excess of isocoproporphyrins in the feces.1

Analysis of UROD activity in erythrocytes can help determine the type of porphyria cutanea tarda. Type I and type III and porphyria cutanea tarda secondary to hepatotoxin exposure have normal levels, whereas type II and the hepatoerythropoietic form have abnormally low levels. Examination of the urine with a Wood lamp reveals coral pink fluorescence due to elimination of porphyrins, and this is another diagnostic clue.

Conditions that need to be ruled out include viral infection with hepatitis B or C or human immunodeficiency virus, iron overload, and hereditary hemochromatosis. Serum alpha fetoprotein level assessment, liver ultrasonography, or even biopsy may be indicated to exclude hepatocellular carcinoma.

TREATMENT

Once secondary causes of porphyria are excluded or treated (eg, advising the patient to avoid alcohol, discontinuing estrogens or iron intake), the next step in management is to reduce the patient’s porphyrin and iron loads. Phlebotomy is the standard way to reduce stores of iron throughout the body and particularly in the liver. It works by interrupting iron-mediated oxidative inhibition of hepatic UROD and the oxidation of hepatic porphyrinogens to porphyrinogens.

This adjustment must be gradual, with about 450 mL of blood removed at intervals of 1 to 2 weeks.7 This improves the cutaneous symptoms progressively, with resolution of vesicles in 2 to 3 months, improvement of skin fragility in 6 to 9 months, and normalization of porphyrin levels in 13 months. The scleroderma, atrophy, hyperpigmentation, and hypertrichosis respond more slowly and may take years to resolve.

Porphyria cutanea tarda can recur, usually with new exposure to risk factors. Treatment by phlebotomy may be stopped when the serum ferritin level has reached low-normal levels; the porphyrin levels may not yet be normal at that point but may continue to decline without additional phlebotomy sessions.

If phlebotomy is contraindicated, alternatives include iron chelation with deferoxamine (Desferal),7 or a low dose of chloroquine (Aralen) (125–250 mg orally twice a week) or hydroxychloroquine (Plaquenil) (100 mg orally twice a week) to avoid acute hepatic damage that may be caused by the release of large amounts of porphyrins that accompany standard dosing levels.

References
  1. Elder GH. Porphyria cutanea tarda. Semin Liver Dis 1998; 18:6775.
  2. Mendez M, Rossetti MV, Del C, Batlle AM, Parera VE. The role of inherited and acquired factors in the development of porphyria cutanea tarda in the Argentinean population. J Am Acad Dermatol 2005; 52:417424.
  3. Bonkovsky HL, Poh-Fitzpatrick M, Pimstone N, et al. Porphyria cutanea tarda, hepatitis C, and HFE gene mutations in North America. Hepatology 1998; 27:16611669.
  4. Ali A, Zein NN. Hepatitis C infection: a systemic disease with extrahepatic manifestations. Cleve Clin J Med 2005; 72:10051016.
  5. Grossman ME, Poh-Fitzpatrick MB. Porphyria cutanea tarda. diagnosis and management. Med Clin North Am 1980; 64:807827.
  6. Cortés JM, Oliva H, Paradinas FJ, Hernandez-Guío C. The pathology of the liver in porphyria cutanea tarda. Histopathology 1980; 4:471485.
  7. Rocchi E, Gibertini P, Cassanelli M, et al. Iron removal therapy in porphyria cutanea tarda: phlebotomy versus slow subcutaneous desferrioxamine infusion. Br J Dermatol 1986; 114:621629.
References
  1. Elder GH. Porphyria cutanea tarda. Semin Liver Dis 1998; 18:6775.
  2. Mendez M, Rossetti MV, Del C, Batlle AM, Parera VE. The role of inherited and acquired factors in the development of porphyria cutanea tarda in the Argentinean population. J Am Acad Dermatol 2005; 52:417424.
  3. Bonkovsky HL, Poh-Fitzpatrick M, Pimstone N, et al. Porphyria cutanea tarda, hepatitis C, and HFE gene mutations in North America. Hepatology 1998; 27:16611669.
  4. Ali A, Zein NN. Hepatitis C infection: a systemic disease with extrahepatic manifestations. Cleve Clin J Med 2005; 72:10051016.
  5. Grossman ME, Poh-Fitzpatrick MB. Porphyria cutanea tarda. diagnosis and management. Med Clin North Am 1980; 64:807827.
  6. Cortés JM, Oliva H, Paradinas FJ, Hernandez-Guío C. The pathology of the liver in porphyria cutanea tarda. Histopathology 1980; 4:471485.
  7. Rocchi E, Gibertini P, Cassanelli M, et al. Iron removal therapy in porphyria cutanea tarda: phlebotomy versus slow subcutaneous desferrioxamine infusion. Br J Dermatol 1986; 114:621629.
Issue
Cleveland Clinic Journal of Medicine - 77(1)
Issue
Cleveland Clinic Journal of Medicine - 77(1)
Page Number
34-36
Page Number
34-36
Publications
Publications
Topics
Article Type
Display Headline
Lesions on the hands, high aminotransferase levels
Display Headline
Lesions on the hands, high aminotransferase levels
Sections
Disallow All Ads
Alternative CME
Article PDF Media

Painful eye with a facial rash

Article Type
Changed
Mon, 03/05/2018 - 08:54
Display Headline
Painful eye with a facial rash

A 75-year-old man presents 4 days after painful cutaneous lesions appeared on the left side of his face, associated with severe ocular pain. Two days before the eruption, he had had an intense headache, which was diagnosed as a tension headache and was treated with oral acetaminophen (Tylenol), but with no improvement.

Figure 1. Typical vesicular rash affects the first trigeminal branch dermatome without trespassing the midline.
He has a history of hypertension and hyperuricemia. No recent changes have been made in his medications. Physical examination shows grouped herpetiform vesicles on an erythematous base confined to the cutaneous surface and innervated by the left first trigeminal branch (Figure 1). Palpation detects regional preauricular and submaxillary lymphadenopathies.

Figure 2. Typical corneal fluorescein stain of dendritic keratitis under cobalt blue light.
Ophthalmologic examination with fluorescein stain shows moderate perilimbal injection and dendritic keratitis (Figure 2).

The remainder of his physical examination is normal. Laboratory tests, including red and white blood cell counts, hemoglobin, and basic metabolic and coagulation tests reveal no abnormalities.

Q: What is your diagnosis?

  • Allergic contact dermatitis
  • Herpes simplex
  • Varicella
  • Ramsay-Hunt syndrome
  • Herpes zoster ophthalmicus and herpetic keratitis

A: Herpes zoster ophthalmicus is the correct diagnosis. It represents a reactivation of the varicella zoster virus.1

Varicella zoster virus, like others of the herpes family, has developed a complex control of virus-host interactions to ensure its survival in humans. It lies dormant in the sensory ganglia and, when reactivated, moves down the neurons and satellite cells along the sensory axons to the skin.1 The reactivation is related to diminished cell-mediated immunity, which occurs as a physiologic part of aging, which is why the elderly tend to be the most often affected. 2 The incidence of herpes zoster varies from 2.2 to 3.4 per 1,000 people per year.3 Its incidence in people over age 80 is about 10 per 1,000 people per year.3

CLINICAL PRESENTATION

Herpes zoster typically presents as a dermatome-grouped vesicular eruption over an erythematous base, accompanied or preceded by local pain. It has two main complications, postherpetic neuralgia and ocular involvement. Postherpetic neuralgia is neuropathic pain that persists or develops after the dermatomal rash has healed.4 Independent predictors of postherpetic neuralgia are older age, severe acute pain, severe rash, a shorter duration of rash before consultation, and ocular involvement.5 It occurs in 36.6% of patients over age 60, and in 47.5% over 70.6 Persistent postherpetic neuralgia has been linked to suicide in patients over 70.7

Ocular infection occurs with involvement of the ophthalmic division of the fifth cranial nerve. Before the antiviral era, it was seen in as many as 50% of patients.8 Hutchinson’s sign is skin lesions on the tip, side, or root of the nose and is an important predictor of ocular involvement.1 Lesions may include folliculopapillar conjunctivitis, episcleritis, scleritis, keratitis (dendritic, pseudodendritic, and interstitial), uveitis, and necrotizing retinitis.

 

 

DIAGNOSIS

The diagnosis of herpes zoster is usually based on clinical observation of the characteristic rash, although viral culture and molecular techniques are available when definitive diagnosis is required. When ophthalmic division is affected and Hutchinson’s sign, unexplained ocular redness with pain, or complaints of visual problems are present, the patient should be referred promptly to an ophthalmologist, because serious visual impairment can occur. The fluorescein dye may show no staining or the typical dendritic keratitis (Figure 2).

TREATMENT

Oral antiviral drugs have made the treatment of zoster possible when, effectively, no treatment existed before. Ideally, an antiviral should be given within 72 hours of symptom onset. Starting treatment as early as possible—especially within 72 hours of onset—has been shown to be effective in alleviating acute pain and in preventing or limiting the duration and severity of postherpetic neuralgia.3

Acyclovir (Zovirax) 800 mg five times a day for 7 days or one of its derivatives—eg, famciclovir (Famvir), penciclovir (Denavir), or valacyclovir (Valtrex)—has been shown to be safe and effective in the treatment of active disease, as well as in preventing or shortening the duration of postherpetic neuralgia.3 It has also been shown to reduce the rate of eye involvement from 50% to 20% or 30%.9 This is why all patients with this dermatomal involvement must be treated.

Second-generation antivirals

Valacyclovir 1,000 mg three times a day and famciclovir 500 mg three times a day seem to be as effective as acyclovir in reducing zoster-associated pain, but their efficacy in reducing eye involvement has not been studied. In clinical practice, however, these second-generation antivirals may be more effective than acyclovir because patients are more likely to comply with the treatment regimen of three rather than five daily doses.

Other considerations

In patients with kidney failure, the non-nephrotoxic antiviral brivudine is preferred, but it is not available in the United States. Therefore, one must use acyclovir or one of the other drugs, carefully adjusting the dose according to the creatinine clearance and making sure the patient is well hydrated.

The efficacy of antiviral treatment that is started more than 72 hours after the onset of skin rash has never been confirmed.

Although the additional effectiveness of acyclovir eye ointment has never been established, topical acyclovir can be considered in cases of dendritic or pseudodendritic keratitis.

References
  1. Liesegang TJ. Herpes zoster ophthalmicus: natural history, risk factors, clinical presentation, and morbidity. Ophthalmology 2008; 115( suppl 2):S3S12.
  2. Opstelten W, Eekhof J, Neven AK, Verheij T. Treatment of herpes zoster. Can Fam Physician 2008; 54:373377.
  3. Opstelten W, Zaal MJ. Managing ophthalmic herpes zoster in primary care. BMJ 2005; 331:147151.
  4. Donahue JG, Choo PW, Manson JE, Platt R. The incidence of herpes zoster. Arch Intern Med 1995; 155:16051609.
  5. Opstelten W, Zuithoff NP, van Essen GA, et al. Predicting postherpetic neuralgia in elderly primary care patients with herpes zoster: prospective prognostic study. Pain 2007; 132( suppl 1):S52S59.
  6. De Morgas JM, Kierland RR. The outcome of patients with herpes zoster. AMA Arch Derm 1957; 75:193196.
  7. Hess TM, Lutz LJ, Nauss LA, Lamer TJ. Treatment of acute herpetic neuralgia. A case report and review of the literature. Minn Med 1990; 73:3740.
  8. Harding SP, Lipton JR, Wells JC. Natural history of herpes zoster ophthalmicus: predictors of postherpetic neuralgia and ocular involvement. Br J Ophthalmol 1987; 71:353358.
  9. Cobo LM, Foulks GN, Liesegang T, et al. Oral acyclovir in the treatment of acute herpes zoster ophthalmicus. Ophthalmology 1986; 93:763770.
Article PDF
Author and Disclosure Information

José Luis Vallejo-García, MD
Department of Ophthalmology, Hospital Universitario La Paz, Madrid, Spain

Sergio Vañó-Galván, MD
Department of Dermatology, Ramón y Cajal Hospital, University of Alcalá, Madrid, Spain

Omar Rayward, MD
Department of Ophthalmology, Hospital Universitario Clínico San Carlos, Madrid, Spain

Paula Moreno-Martin, MD
Department of Ophthalmology, La Princesa University Hospital, Madrid, Spain

Address: José Luis Vallejo-García, MD, Paseo de la Castellana Nº 261, 28046 Madrid, Spain; e-mail jvallejogarcia@gmail.com

Issue
Cleveland Clinic Journal of Medicine - 76(7)
Publications
Topics
Page Number
410-412
Sections
Author and Disclosure Information

José Luis Vallejo-García, MD
Department of Ophthalmology, Hospital Universitario La Paz, Madrid, Spain

Sergio Vañó-Galván, MD
Department of Dermatology, Ramón y Cajal Hospital, University of Alcalá, Madrid, Spain

Omar Rayward, MD
Department of Ophthalmology, Hospital Universitario Clínico San Carlos, Madrid, Spain

Paula Moreno-Martin, MD
Department of Ophthalmology, La Princesa University Hospital, Madrid, Spain

Address: José Luis Vallejo-García, MD, Paseo de la Castellana Nº 261, 28046 Madrid, Spain; e-mail jvallejogarcia@gmail.com

Author and Disclosure Information

José Luis Vallejo-García, MD
Department of Ophthalmology, Hospital Universitario La Paz, Madrid, Spain

Sergio Vañó-Galván, MD
Department of Dermatology, Ramón y Cajal Hospital, University of Alcalá, Madrid, Spain

Omar Rayward, MD
Department of Ophthalmology, Hospital Universitario Clínico San Carlos, Madrid, Spain

Paula Moreno-Martin, MD
Department of Ophthalmology, La Princesa University Hospital, Madrid, Spain

Address: José Luis Vallejo-García, MD, Paseo de la Castellana Nº 261, 28046 Madrid, Spain; e-mail jvallejogarcia@gmail.com

Article PDF
Article PDF

A 75-year-old man presents 4 days after painful cutaneous lesions appeared on the left side of his face, associated with severe ocular pain. Two days before the eruption, he had had an intense headache, which was diagnosed as a tension headache and was treated with oral acetaminophen (Tylenol), but with no improvement.

Figure 1. Typical vesicular rash affects the first trigeminal branch dermatome without trespassing the midline.
He has a history of hypertension and hyperuricemia. No recent changes have been made in his medications. Physical examination shows grouped herpetiform vesicles on an erythematous base confined to the cutaneous surface and innervated by the left first trigeminal branch (Figure 1). Palpation detects regional preauricular and submaxillary lymphadenopathies.

Figure 2. Typical corneal fluorescein stain of dendritic keratitis under cobalt blue light.
Ophthalmologic examination with fluorescein stain shows moderate perilimbal injection and dendritic keratitis (Figure 2).

The remainder of his physical examination is normal. Laboratory tests, including red and white blood cell counts, hemoglobin, and basic metabolic and coagulation tests reveal no abnormalities.

Q: What is your diagnosis?

  • Allergic contact dermatitis
  • Herpes simplex
  • Varicella
  • Ramsay-Hunt syndrome
  • Herpes zoster ophthalmicus and herpetic keratitis

A: Herpes zoster ophthalmicus is the correct diagnosis. It represents a reactivation of the varicella zoster virus.1

Varicella zoster virus, like others of the herpes family, has developed a complex control of virus-host interactions to ensure its survival in humans. It lies dormant in the sensory ganglia and, when reactivated, moves down the neurons and satellite cells along the sensory axons to the skin.1 The reactivation is related to diminished cell-mediated immunity, which occurs as a physiologic part of aging, which is why the elderly tend to be the most often affected. 2 The incidence of herpes zoster varies from 2.2 to 3.4 per 1,000 people per year.3 Its incidence in people over age 80 is about 10 per 1,000 people per year.3

CLINICAL PRESENTATION

Herpes zoster typically presents as a dermatome-grouped vesicular eruption over an erythematous base, accompanied or preceded by local pain. It has two main complications, postherpetic neuralgia and ocular involvement. Postherpetic neuralgia is neuropathic pain that persists or develops after the dermatomal rash has healed.4 Independent predictors of postherpetic neuralgia are older age, severe acute pain, severe rash, a shorter duration of rash before consultation, and ocular involvement.5 It occurs in 36.6% of patients over age 60, and in 47.5% over 70.6 Persistent postherpetic neuralgia has been linked to suicide in patients over 70.7

Ocular infection occurs with involvement of the ophthalmic division of the fifth cranial nerve. Before the antiviral era, it was seen in as many as 50% of patients.8 Hutchinson’s sign is skin lesions on the tip, side, or root of the nose and is an important predictor of ocular involvement.1 Lesions may include folliculopapillar conjunctivitis, episcleritis, scleritis, keratitis (dendritic, pseudodendritic, and interstitial), uveitis, and necrotizing retinitis.

 

 

DIAGNOSIS

The diagnosis of herpes zoster is usually based on clinical observation of the characteristic rash, although viral culture and molecular techniques are available when definitive diagnosis is required. When ophthalmic division is affected and Hutchinson’s sign, unexplained ocular redness with pain, or complaints of visual problems are present, the patient should be referred promptly to an ophthalmologist, because serious visual impairment can occur. The fluorescein dye may show no staining or the typical dendritic keratitis (Figure 2).

TREATMENT

Oral antiviral drugs have made the treatment of zoster possible when, effectively, no treatment existed before. Ideally, an antiviral should be given within 72 hours of symptom onset. Starting treatment as early as possible—especially within 72 hours of onset—has been shown to be effective in alleviating acute pain and in preventing or limiting the duration and severity of postherpetic neuralgia.3

Acyclovir (Zovirax) 800 mg five times a day for 7 days or one of its derivatives—eg, famciclovir (Famvir), penciclovir (Denavir), or valacyclovir (Valtrex)—has been shown to be safe and effective in the treatment of active disease, as well as in preventing or shortening the duration of postherpetic neuralgia.3 It has also been shown to reduce the rate of eye involvement from 50% to 20% or 30%.9 This is why all patients with this dermatomal involvement must be treated.

Second-generation antivirals

Valacyclovir 1,000 mg three times a day and famciclovir 500 mg three times a day seem to be as effective as acyclovir in reducing zoster-associated pain, but their efficacy in reducing eye involvement has not been studied. In clinical practice, however, these second-generation antivirals may be more effective than acyclovir because patients are more likely to comply with the treatment regimen of three rather than five daily doses.

Other considerations

In patients with kidney failure, the non-nephrotoxic antiviral brivudine is preferred, but it is not available in the United States. Therefore, one must use acyclovir or one of the other drugs, carefully adjusting the dose according to the creatinine clearance and making sure the patient is well hydrated.

The efficacy of antiviral treatment that is started more than 72 hours after the onset of skin rash has never been confirmed.

Although the additional effectiveness of acyclovir eye ointment has never been established, topical acyclovir can be considered in cases of dendritic or pseudodendritic keratitis.

A 75-year-old man presents 4 days after painful cutaneous lesions appeared on the left side of his face, associated with severe ocular pain. Two days before the eruption, he had had an intense headache, which was diagnosed as a tension headache and was treated with oral acetaminophen (Tylenol), but with no improvement.

Figure 1. Typical vesicular rash affects the first trigeminal branch dermatome without trespassing the midline.
He has a history of hypertension and hyperuricemia. No recent changes have been made in his medications. Physical examination shows grouped herpetiform vesicles on an erythematous base confined to the cutaneous surface and innervated by the left first trigeminal branch (Figure 1). Palpation detects regional preauricular and submaxillary lymphadenopathies.

Figure 2. Typical corneal fluorescein stain of dendritic keratitis under cobalt blue light.
Ophthalmologic examination with fluorescein stain shows moderate perilimbal injection and dendritic keratitis (Figure 2).

The remainder of his physical examination is normal. Laboratory tests, including red and white blood cell counts, hemoglobin, and basic metabolic and coagulation tests reveal no abnormalities.

Q: What is your diagnosis?

  • Allergic contact dermatitis
  • Herpes simplex
  • Varicella
  • Ramsay-Hunt syndrome
  • Herpes zoster ophthalmicus and herpetic keratitis

A: Herpes zoster ophthalmicus is the correct diagnosis. It represents a reactivation of the varicella zoster virus.1

Varicella zoster virus, like others of the herpes family, has developed a complex control of virus-host interactions to ensure its survival in humans. It lies dormant in the sensory ganglia and, when reactivated, moves down the neurons and satellite cells along the sensory axons to the skin.1 The reactivation is related to diminished cell-mediated immunity, which occurs as a physiologic part of aging, which is why the elderly tend to be the most often affected. 2 The incidence of herpes zoster varies from 2.2 to 3.4 per 1,000 people per year.3 Its incidence in people over age 80 is about 10 per 1,000 people per year.3

CLINICAL PRESENTATION

Herpes zoster typically presents as a dermatome-grouped vesicular eruption over an erythematous base, accompanied or preceded by local pain. It has two main complications, postherpetic neuralgia and ocular involvement. Postherpetic neuralgia is neuropathic pain that persists or develops after the dermatomal rash has healed.4 Independent predictors of postherpetic neuralgia are older age, severe acute pain, severe rash, a shorter duration of rash before consultation, and ocular involvement.5 It occurs in 36.6% of patients over age 60, and in 47.5% over 70.6 Persistent postherpetic neuralgia has been linked to suicide in patients over 70.7

Ocular infection occurs with involvement of the ophthalmic division of the fifth cranial nerve. Before the antiviral era, it was seen in as many as 50% of patients.8 Hutchinson’s sign is skin lesions on the tip, side, or root of the nose and is an important predictor of ocular involvement.1 Lesions may include folliculopapillar conjunctivitis, episcleritis, scleritis, keratitis (dendritic, pseudodendritic, and interstitial), uveitis, and necrotizing retinitis.

 

 

DIAGNOSIS

The diagnosis of herpes zoster is usually based on clinical observation of the characteristic rash, although viral culture and molecular techniques are available when definitive diagnosis is required. When ophthalmic division is affected and Hutchinson’s sign, unexplained ocular redness with pain, or complaints of visual problems are present, the patient should be referred promptly to an ophthalmologist, because serious visual impairment can occur. The fluorescein dye may show no staining or the typical dendritic keratitis (Figure 2).

TREATMENT

Oral antiviral drugs have made the treatment of zoster possible when, effectively, no treatment existed before. Ideally, an antiviral should be given within 72 hours of symptom onset. Starting treatment as early as possible—especially within 72 hours of onset—has been shown to be effective in alleviating acute pain and in preventing or limiting the duration and severity of postherpetic neuralgia.3

Acyclovir (Zovirax) 800 mg five times a day for 7 days or one of its derivatives—eg, famciclovir (Famvir), penciclovir (Denavir), or valacyclovir (Valtrex)—has been shown to be safe and effective in the treatment of active disease, as well as in preventing or shortening the duration of postherpetic neuralgia.3 It has also been shown to reduce the rate of eye involvement from 50% to 20% or 30%.9 This is why all patients with this dermatomal involvement must be treated.

Second-generation antivirals

Valacyclovir 1,000 mg three times a day and famciclovir 500 mg three times a day seem to be as effective as acyclovir in reducing zoster-associated pain, but their efficacy in reducing eye involvement has not been studied. In clinical practice, however, these second-generation antivirals may be more effective than acyclovir because patients are more likely to comply with the treatment regimen of three rather than five daily doses.

Other considerations

In patients with kidney failure, the non-nephrotoxic antiviral brivudine is preferred, but it is not available in the United States. Therefore, one must use acyclovir or one of the other drugs, carefully adjusting the dose according to the creatinine clearance and making sure the patient is well hydrated.

The efficacy of antiviral treatment that is started more than 72 hours after the onset of skin rash has never been confirmed.

Although the additional effectiveness of acyclovir eye ointment has never been established, topical acyclovir can be considered in cases of dendritic or pseudodendritic keratitis.

References
  1. Liesegang TJ. Herpes zoster ophthalmicus: natural history, risk factors, clinical presentation, and morbidity. Ophthalmology 2008; 115( suppl 2):S3S12.
  2. Opstelten W, Eekhof J, Neven AK, Verheij T. Treatment of herpes zoster. Can Fam Physician 2008; 54:373377.
  3. Opstelten W, Zaal MJ. Managing ophthalmic herpes zoster in primary care. BMJ 2005; 331:147151.
  4. Donahue JG, Choo PW, Manson JE, Platt R. The incidence of herpes zoster. Arch Intern Med 1995; 155:16051609.
  5. Opstelten W, Zuithoff NP, van Essen GA, et al. Predicting postherpetic neuralgia in elderly primary care patients with herpes zoster: prospective prognostic study. Pain 2007; 132( suppl 1):S52S59.
  6. De Morgas JM, Kierland RR. The outcome of patients with herpes zoster. AMA Arch Derm 1957; 75:193196.
  7. Hess TM, Lutz LJ, Nauss LA, Lamer TJ. Treatment of acute herpetic neuralgia. A case report and review of the literature. Minn Med 1990; 73:3740.
  8. Harding SP, Lipton JR, Wells JC. Natural history of herpes zoster ophthalmicus: predictors of postherpetic neuralgia and ocular involvement. Br J Ophthalmol 1987; 71:353358.
  9. Cobo LM, Foulks GN, Liesegang T, et al. Oral acyclovir in the treatment of acute herpes zoster ophthalmicus. Ophthalmology 1986; 93:763770.
References
  1. Liesegang TJ. Herpes zoster ophthalmicus: natural history, risk factors, clinical presentation, and morbidity. Ophthalmology 2008; 115( suppl 2):S3S12.
  2. Opstelten W, Eekhof J, Neven AK, Verheij T. Treatment of herpes zoster. Can Fam Physician 2008; 54:373377.
  3. Opstelten W, Zaal MJ. Managing ophthalmic herpes zoster in primary care. BMJ 2005; 331:147151.
  4. Donahue JG, Choo PW, Manson JE, Platt R. The incidence of herpes zoster. Arch Intern Med 1995; 155:16051609.
  5. Opstelten W, Zuithoff NP, van Essen GA, et al. Predicting postherpetic neuralgia in elderly primary care patients with herpes zoster: prospective prognostic study. Pain 2007; 132( suppl 1):S52S59.
  6. De Morgas JM, Kierland RR. The outcome of patients with herpes zoster. AMA Arch Derm 1957; 75:193196.
  7. Hess TM, Lutz LJ, Nauss LA, Lamer TJ. Treatment of acute herpetic neuralgia. A case report and review of the literature. Minn Med 1990; 73:3740.
  8. Harding SP, Lipton JR, Wells JC. Natural history of herpes zoster ophthalmicus: predictors of postherpetic neuralgia and ocular involvement. Br J Ophthalmol 1987; 71:353358.
  9. Cobo LM, Foulks GN, Liesegang T, et al. Oral acyclovir in the treatment of acute herpes zoster ophthalmicus. Ophthalmology 1986; 93:763770.
Issue
Cleveland Clinic Journal of Medicine - 76(7)
Issue
Cleveland Clinic Journal of Medicine - 76(7)
Page Number
410-412
Page Number
410-412
Publications
Publications
Topics
Article Type
Display Headline
Painful eye with a facial rash
Display Headline
Painful eye with a facial rash
Sections
Disallow All Ads
Alternative CME
Article PDF Media

A facial cutaneous horn

Article Type
Changed
Fri, 02/09/2018 - 08:39
Display Headline
A facial cutaneous horn

Figure 1. Grey-colored cutaneous horn over a pink-colored nodule on the left cheek.
A healthy 84-year-old woman is referred to the dermatology department with a 10-month history of an asymptomatic lesion on her left cheek that has grown progressively. The lesion appeared spontaneously and was not associated with trauma.

Figure 2. The keratotic component of the lesion has a hard consistency on palpation.
On examination, there is a 2-cm-long, keratotic, gray cutaneous horn over a pink nodule on the left cheek (Figures 1 and 2). There is no lymphadenopathy, and the patient has no other cutaneous abnormalities.

The patient says she has had intense sunlight exposure during her life, due to agricultural work.

Q: What is the most likely diagnosis?

  • Basal cell carcinoma
  • Cutaneous leishmaniasis
  • Squamous cell carcinoma
  • Viral wart
  • Pyoderma gangrenosum

A: Squamous cell carcinoma is the most likely diagnosis. It is the second most common form of skin cancer (after basal cell carcinoma) that arises on the sun-exposed skin of elderly individuals.

Basal cell carcinoma usually presents as a slow-growing, pearly nodule with telangiectasias over the surface, but without a keratotic component. Cutaneous leishmaniasis is caused by a protozoan and is transmitted by the bite of a sandfly. It presents as a crusted papule or ulcer at the site of the bite, only rarely presenting as a cutaneous horn. Viral warts may present as horns (filiform warts), but the horn is slender and is usually around the lips, eyelids, or nares. Although warts can occur at any age, they are more frequently seen among school-aged children, and they peak between ages 12 and 16. Pyoderma gangrenosum is considered a reactive dermatosis sometimes secondary to a systemic disease such as inflammatory bowel disease. It usually presents as a small, red papule or pustule that progresses to a larger ulcerative lesion.

SQUAMOUS CELL CARCINOMA: CAUSES, RISK FACTORS

Exposure to ultraviolet radiation is the most common cause of squamous cell carcinoma. Other risk factors include exposure to ionizing radiation, human papilloma virus, thermal injury, burns, immunosuppression, exposure to chemical carcinogens, and diseases such as xeroderma pigmentosum, oculocutaneous albinism, and junctional epidermolysis bullosa.1,2 Organ transplantation is another significant risk factor, and cutaneous squamous cell carcinoma is the most common malignancy in renal transplant recipients.3

People born in areas that receive high amounts of ultraviolet radiation from the sun have a risk of squamous cell carcinoma three times higher than people who move to those areas in adulthood.1 Occupational exposure to ultraviolet radiation is also implicated in this type of cancer.

CLINICAL APPEARANCE

Clinically, squamous cell carcinoma of the skin may manifest as a variety of primary morphologies with or without associated symptoms. The characteristic invasive squamous cell carcinoma is a raised, firm, pink or flesh-colored keratotic papule or plaque arising on sun-exposed skin. Approximately 70% of all squamous cell carcinomas occur on the head and neck, with an additional 15% found on the upper extremities. Surface changes may include scaling, ulceration, crusting, or the presence of a cutaneous horn. Squamous cell carcinoma can also present as an eczematous patch that does not resolve with emollient or topical steroids. Less commonly, it may manifest as a pink cutaneous nodule with no overlying surface changes. Often, the patient has a background of severely sun-damaged skin, including solar elastosis, mottled dyspigmentation, telangiectasia, and multiple actinic keratoses.

MAKING THE DIAGNOSIS

Skin biopsy usually confirms the clinical diagnosis. Histologic examination shows neoplastic proliferation of atypical keratinocytes extending into the dermis, with large hyperchromatic and pleomorphic nuclei.

Imaging is not routinely indicated for diagnosing cutaneous squamous cell carcinoma. However, radiologic imaging should be obtained in patients with regional lymphadenopathy or neurologic symptoms suggestive of perineural involvement.

Other lesions found at the base of a cutaneous horn include actinic keratosis, keratoacanthoma, basal cell carcinoma, and benign lesions such as seborrheic keratosis and verruca vulgaris.4,5

MANAGEMENT

Most cutaneous squamous cell carcinomas are easily treated, and the rate of cure is high. Treatment options include topical chemotherapy, cryosurgery, electrodesiccation and curettage, surgical excision, Mohs micrographic surgery, radiotherapy, and photodynamic therapy. Fractionated radiation treatment may be preferred for patients who are unable to tolerate surgery or who have inoperable tumors, and it may provide favorable functional and cosmetic results.1

Photodynamic therapy and topical chemotherapy with imiquimod are currently not recommended for invasive squamous cell carcinoma.

Low-risk tumors are usually cured with surgical therapy. However, patients who develop one squamous cell carcinoma have a 40% risk of developing additional squamous cell carcinomas within the next 2 years. This risk is likely even greater as more time elapses. Thus, patients with a history of this cancer should be evaluated with a complete skin examination every 6 to 12 months.

OUR PATIENT’S TREATMENT

Our patient underwent total surgical excision of the lesion with a safety margin of 0.5 cm. Histologic results confirmed the diagnosis of squamous cell carcinoma with tumor-free margins. No clinical relapse was observed after 1 year of follow-up.

References
  1. Alam M, Ratner D. Cutaneous squamous-cell carcinoma. N Engl J Med 2001; 344:975983.
  2. Mallipeddi R, Keane FM, Mcgrath JA, Mayou BJ, Eady RA. Increased risk of squamous cell carcinoma in junctional epidermolysis bullosa. J Eur Acad Dermatol Venereol 2004; 18:521526.
  3. Rubel JR, Milford EL, Abdi R. Cutaneous neoplasms in renal transplant recipients. Eur J Dermatol 2002; 12:532535.
  4. Aydogan K, Ozbek S, Balaban Adim S, Tokgöz N. Irritated seborrhoeic keratosis presenting as a cutaneous horn. J Eur Acad Dermatol Venereol 2006; 20:626628.
  5. Vañó-Galván S, Sanchez-Olaso A. Images in clinical medicine. Squamous-cell carcinoma manifesting as a cutaneous horn. N Engl J Med 2008; 359:e10.
Article PDF
Author and Disclosure Information

Sergio Vañó-Galván, MD
Department of Dermatology, Ramón y Cajal University Hospital, University of Alcalá, Madrid, Spain

Alicia Marqués, MD
Department of Dermatology, Ramón y Cajal University Hospital, University of Alcalá, Madrid, Spain

Ernesto Muñoz-Zato, MD
Department of Dermatology, Ramón y Cajal University Hospital, University of Alcalá, Madrid, Spain

Pedro Jaén, MD, PhD
Department of Dermatology, Ramón y Cajal University Hospital, University of Alcalá, Madrid, Spain

Address: Sergio Vañó-Galván, MD, Department of Dermatology, Ramón y Cajal Hospital, University of Alcalá, Carretera Colmenar Viejo, km 9.100, 28034 Madrid, Spain; sergiovano@yahoo.es

Issue
Cleveland Clinic Journal of Medicine - 76(2)
Publications
Topics
Page Number
92-95
Sections
Author and Disclosure Information

Sergio Vañó-Galván, MD
Department of Dermatology, Ramón y Cajal University Hospital, University of Alcalá, Madrid, Spain

Alicia Marqués, MD
Department of Dermatology, Ramón y Cajal University Hospital, University of Alcalá, Madrid, Spain

Ernesto Muñoz-Zato, MD
Department of Dermatology, Ramón y Cajal University Hospital, University of Alcalá, Madrid, Spain

Pedro Jaén, MD, PhD
Department of Dermatology, Ramón y Cajal University Hospital, University of Alcalá, Madrid, Spain

Address: Sergio Vañó-Galván, MD, Department of Dermatology, Ramón y Cajal Hospital, University of Alcalá, Carretera Colmenar Viejo, km 9.100, 28034 Madrid, Spain; sergiovano@yahoo.es

Author and Disclosure Information

Sergio Vañó-Galván, MD
Department of Dermatology, Ramón y Cajal University Hospital, University of Alcalá, Madrid, Spain

Alicia Marqués, MD
Department of Dermatology, Ramón y Cajal University Hospital, University of Alcalá, Madrid, Spain

Ernesto Muñoz-Zato, MD
Department of Dermatology, Ramón y Cajal University Hospital, University of Alcalá, Madrid, Spain

Pedro Jaén, MD, PhD
Department of Dermatology, Ramón y Cajal University Hospital, University of Alcalá, Madrid, Spain

Address: Sergio Vañó-Galván, MD, Department of Dermatology, Ramón y Cajal Hospital, University of Alcalá, Carretera Colmenar Viejo, km 9.100, 28034 Madrid, Spain; sergiovano@yahoo.es

Article PDF
Article PDF

Figure 1. Grey-colored cutaneous horn over a pink-colored nodule on the left cheek.
A healthy 84-year-old woman is referred to the dermatology department with a 10-month history of an asymptomatic lesion on her left cheek that has grown progressively. The lesion appeared spontaneously and was not associated with trauma.

Figure 2. The keratotic component of the lesion has a hard consistency on palpation.
On examination, there is a 2-cm-long, keratotic, gray cutaneous horn over a pink nodule on the left cheek (Figures 1 and 2). There is no lymphadenopathy, and the patient has no other cutaneous abnormalities.

The patient says she has had intense sunlight exposure during her life, due to agricultural work.

Q: What is the most likely diagnosis?

  • Basal cell carcinoma
  • Cutaneous leishmaniasis
  • Squamous cell carcinoma
  • Viral wart
  • Pyoderma gangrenosum

A: Squamous cell carcinoma is the most likely diagnosis. It is the second most common form of skin cancer (after basal cell carcinoma) that arises on the sun-exposed skin of elderly individuals.

Basal cell carcinoma usually presents as a slow-growing, pearly nodule with telangiectasias over the surface, but without a keratotic component. Cutaneous leishmaniasis is caused by a protozoan and is transmitted by the bite of a sandfly. It presents as a crusted papule or ulcer at the site of the bite, only rarely presenting as a cutaneous horn. Viral warts may present as horns (filiform warts), but the horn is slender and is usually around the lips, eyelids, or nares. Although warts can occur at any age, they are more frequently seen among school-aged children, and they peak between ages 12 and 16. Pyoderma gangrenosum is considered a reactive dermatosis sometimes secondary to a systemic disease such as inflammatory bowel disease. It usually presents as a small, red papule or pustule that progresses to a larger ulcerative lesion.

SQUAMOUS CELL CARCINOMA: CAUSES, RISK FACTORS

Exposure to ultraviolet radiation is the most common cause of squamous cell carcinoma. Other risk factors include exposure to ionizing radiation, human papilloma virus, thermal injury, burns, immunosuppression, exposure to chemical carcinogens, and diseases such as xeroderma pigmentosum, oculocutaneous albinism, and junctional epidermolysis bullosa.1,2 Organ transplantation is another significant risk factor, and cutaneous squamous cell carcinoma is the most common malignancy in renal transplant recipients.3

People born in areas that receive high amounts of ultraviolet radiation from the sun have a risk of squamous cell carcinoma three times higher than people who move to those areas in adulthood.1 Occupational exposure to ultraviolet radiation is also implicated in this type of cancer.

CLINICAL APPEARANCE

Clinically, squamous cell carcinoma of the skin may manifest as a variety of primary morphologies with or without associated symptoms. The characteristic invasive squamous cell carcinoma is a raised, firm, pink or flesh-colored keratotic papule or plaque arising on sun-exposed skin. Approximately 70% of all squamous cell carcinomas occur on the head and neck, with an additional 15% found on the upper extremities. Surface changes may include scaling, ulceration, crusting, or the presence of a cutaneous horn. Squamous cell carcinoma can also present as an eczematous patch that does not resolve with emollient or topical steroids. Less commonly, it may manifest as a pink cutaneous nodule with no overlying surface changes. Often, the patient has a background of severely sun-damaged skin, including solar elastosis, mottled dyspigmentation, telangiectasia, and multiple actinic keratoses.

MAKING THE DIAGNOSIS

Skin biopsy usually confirms the clinical diagnosis. Histologic examination shows neoplastic proliferation of atypical keratinocytes extending into the dermis, with large hyperchromatic and pleomorphic nuclei.

Imaging is not routinely indicated for diagnosing cutaneous squamous cell carcinoma. However, radiologic imaging should be obtained in patients with regional lymphadenopathy or neurologic symptoms suggestive of perineural involvement.

Other lesions found at the base of a cutaneous horn include actinic keratosis, keratoacanthoma, basal cell carcinoma, and benign lesions such as seborrheic keratosis and verruca vulgaris.4,5

MANAGEMENT

Most cutaneous squamous cell carcinomas are easily treated, and the rate of cure is high. Treatment options include topical chemotherapy, cryosurgery, electrodesiccation and curettage, surgical excision, Mohs micrographic surgery, radiotherapy, and photodynamic therapy. Fractionated radiation treatment may be preferred for patients who are unable to tolerate surgery or who have inoperable tumors, and it may provide favorable functional and cosmetic results.1

Photodynamic therapy and topical chemotherapy with imiquimod are currently not recommended for invasive squamous cell carcinoma.

Low-risk tumors are usually cured with surgical therapy. However, patients who develop one squamous cell carcinoma have a 40% risk of developing additional squamous cell carcinomas within the next 2 years. This risk is likely even greater as more time elapses. Thus, patients with a history of this cancer should be evaluated with a complete skin examination every 6 to 12 months.

OUR PATIENT’S TREATMENT

Our patient underwent total surgical excision of the lesion with a safety margin of 0.5 cm. Histologic results confirmed the diagnosis of squamous cell carcinoma with tumor-free margins. No clinical relapse was observed after 1 year of follow-up.

Figure 1. Grey-colored cutaneous horn over a pink-colored nodule on the left cheek.
A healthy 84-year-old woman is referred to the dermatology department with a 10-month history of an asymptomatic lesion on her left cheek that has grown progressively. The lesion appeared spontaneously and was not associated with trauma.

Figure 2. The keratotic component of the lesion has a hard consistency on palpation.
On examination, there is a 2-cm-long, keratotic, gray cutaneous horn over a pink nodule on the left cheek (Figures 1 and 2). There is no lymphadenopathy, and the patient has no other cutaneous abnormalities.

The patient says she has had intense sunlight exposure during her life, due to agricultural work.

Q: What is the most likely diagnosis?

  • Basal cell carcinoma
  • Cutaneous leishmaniasis
  • Squamous cell carcinoma
  • Viral wart
  • Pyoderma gangrenosum

A: Squamous cell carcinoma is the most likely diagnosis. It is the second most common form of skin cancer (after basal cell carcinoma) that arises on the sun-exposed skin of elderly individuals.

Basal cell carcinoma usually presents as a slow-growing, pearly nodule with telangiectasias over the surface, but without a keratotic component. Cutaneous leishmaniasis is caused by a protozoan and is transmitted by the bite of a sandfly. It presents as a crusted papule or ulcer at the site of the bite, only rarely presenting as a cutaneous horn. Viral warts may present as horns (filiform warts), but the horn is slender and is usually around the lips, eyelids, or nares. Although warts can occur at any age, they are more frequently seen among school-aged children, and they peak between ages 12 and 16. Pyoderma gangrenosum is considered a reactive dermatosis sometimes secondary to a systemic disease such as inflammatory bowel disease. It usually presents as a small, red papule or pustule that progresses to a larger ulcerative lesion.

SQUAMOUS CELL CARCINOMA: CAUSES, RISK FACTORS

Exposure to ultraviolet radiation is the most common cause of squamous cell carcinoma. Other risk factors include exposure to ionizing radiation, human papilloma virus, thermal injury, burns, immunosuppression, exposure to chemical carcinogens, and diseases such as xeroderma pigmentosum, oculocutaneous albinism, and junctional epidermolysis bullosa.1,2 Organ transplantation is another significant risk factor, and cutaneous squamous cell carcinoma is the most common malignancy in renal transplant recipients.3

People born in areas that receive high amounts of ultraviolet radiation from the sun have a risk of squamous cell carcinoma three times higher than people who move to those areas in adulthood.1 Occupational exposure to ultraviolet radiation is also implicated in this type of cancer.

CLINICAL APPEARANCE

Clinically, squamous cell carcinoma of the skin may manifest as a variety of primary morphologies with or without associated symptoms. The characteristic invasive squamous cell carcinoma is a raised, firm, pink or flesh-colored keratotic papule or plaque arising on sun-exposed skin. Approximately 70% of all squamous cell carcinomas occur on the head and neck, with an additional 15% found on the upper extremities. Surface changes may include scaling, ulceration, crusting, or the presence of a cutaneous horn. Squamous cell carcinoma can also present as an eczematous patch that does not resolve with emollient or topical steroids. Less commonly, it may manifest as a pink cutaneous nodule with no overlying surface changes. Often, the patient has a background of severely sun-damaged skin, including solar elastosis, mottled dyspigmentation, telangiectasia, and multiple actinic keratoses.

MAKING THE DIAGNOSIS

Skin biopsy usually confirms the clinical diagnosis. Histologic examination shows neoplastic proliferation of atypical keratinocytes extending into the dermis, with large hyperchromatic and pleomorphic nuclei.

Imaging is not routinely indicated for diagnosing cutaneous squamous cell carcinoma. However, radiologic imaging should be obtained in patients with regional lymphadenopathy or neurologic symptoms suggestive of perineural involvement.

Other lesions found at the base of a cutaneous horn include actinic keratosis, keratoacanthoma, basal cell carcinoma, and benign lesions such as seborrheic keratosis and verruca vulgaris.4,5

MANAGEMENT

Most cutaneous squamous cell carcinomas are easily treated, and the rate of cure is high. Treatment options include topical chemotherapy, cryosurgery, electrodesiccation and curettage, surgical excision, Mohs micrographic surgery, radiotherapy, and photodynamic therapy. Fractionated radiation treatment may be preferred for patients who are unable to tolerate surgery or who have inoperable tumors, and it may provide favorable functional and cosmetic results.1

Photodynamic therapy and topical chemotherapy with imiquimod are currently not recommended for invasive squamous cell carcinoma.

Low-risk tumors are usually cured with surgical therapy. However, patients who develop one squamous cell carcinoma have a 40% risk of developing additional squamous cell carcinomas within the next 2 years. This risk is likely even greater as more time elapses. Thus, patients with a history of this cancer should be evaluated with a complete skin examination every 6 to 12 months.

OUR PATIENT’S TREATMENT

Our patient underwent total surgical excision of the lesion with a safety margin of 0.5 cm. Histologic results confirmed the diagnosis of squamous cell carcinoma with tumor-free margins. No clinical relapse was observed after 1 year of follow-up.

References
  1. Alam M, Ratner D. Cutaneous squamous-cell carcinoma. N Engl J Med 2001; 344:975983.
  2. Mallipeddi R, Keane FM, Mcgrath JA, Mayou BJ, Eady RA. Increased risk of squamous cell carcinoma in junctional epidermolysis bullosa. J Eur Acad Dermatol Venereol 2004; 18:521526.
  3. Rubel JR, Milford EL, Abdi R. Cutaneous neoplasms in renal transplant recipients. Eur J Dermatol 2002; 12:532535.
  4. Aydogan K, Ozbek S, Balaban Adim S, Tokgöz N. Irritated seborrhoeic keratosis presenting as a cutaneous horn. J Eur Acad Dermatol Venereol 2006; 20:626628.
  5. Vañó-Galván S, Sanchez-Olaso A. Images in clinical medicine. Squamous-cell carcinoma manifesting as a cutaneous horn. N Engl J Med 2008; 359:e10.
References
  1. Alam M, Ratner D. Cutaneous squamous-cell carcinoma. N Engl J Med 2001; 344:975983.
  2. Mallipeddi R, Keane FM, Mcgrath JA, Mayou BJ, Eady RA. Increased risk of squamous cell carcinoma in junctional epidermolysis bullosa. J Eur Acad Dermatol Venereol 2004; 18:521526.
  3. Rubel JR, Milford EL, Abdi R. Cutaneous neoplasms in renal transplant recipients. Eur J Dermatol 2002; 12:532535.
  4. Aydogan K, Ozbek S, Balaban Adim S, Tokgöz N. Irritated seborrhoeic keratosis presenting as a cutaneous horn. J Eur Acad Dermatol Venereol 2006; 20:626628.
  5. Vañó-Galván S, Sanchez-Olaso A. Images in clinical medicine. Squamous-cell carcinoma manifesting as a cutaneous horn. N Engl J Med 2008; 359:e10.
Issue
Cleveland Clinic Journal of Medicine - 76(2)
Issue
Cleveland Clinic Journal of Medicine - 76(2)
Page Number
92-95
Page Number
92-95
Publications
Publications
Topics
Article Type
Display Headline
A facial cutaneous horn
Display Headline
A facial cutaneous horn
Sections
Disallow All Ads
Alternative CME
Article PDF Media

A persistently swollen lip

Article Type
Changed
Wed, 01/31/2018 - 09:13
Display Headline
A persistently swollen lip

A 44-year-old man is referred for evaluation of asymptomatic swelling of the lower lip that has persisted for 10 months. He has been treated unsuccessfully with oral antihistamines for suspected chronic angioedema. He has no other symptoms and appears to be well otherwise. He has no history of applied irritants or local trauma, and his medical history is unremarkable.

Figure 1.
Physical examination shows a swollen and indurated lower lip (Figure 1). The tongue and gums are normal, and the ophthalmologic evaluation reveals no abnormalities. No facial paralysis is observed.

Results of the laboratory evaluation, including serum angiotensin-converting enzyme level, are normal. Patch tests to detect contact sensitivity to food additives are negative. Biopsy of the affected lip reveals dense infiltrate of the submucosal connective tissue with focal nonnecrotizing granulomas. Imaging and endoscopic studies show no evidence of sarcoidosis or Crohn disease.

Q: Given what we know so far, which of the following is the most likely diagnosis of the persistent lip swelling?

  • Melkersson-Rosenthal syndrome
  • Amyloidosis
  • Quincke edema
  • Cheilitis granulomatosa
  • Cutaneous tuberculosis

A: From what we know so far, the correct answer is cheilitis granulomatosa. While this rare condition may be a feature of Melkersson-Rosenthal syndrome and amyloidosis, at this point in the evaluation these have not been confirmed. Quincke edema (ie, angioedema) is unlikely, given the ineffectiveness of previous treatment with oral antihistamines. Cutaneous tuberculosis usually presents as “lupus vulgaris,” which is characterized by solitary, small, sharply marginated, red-brown papules of gelatinous consistency (“apple-jelly nodules”), mainly on the head and neck.

Cheilitis granulomatosa is a rare inflammatory disorder1 that primarily affects young adults. Its key feature is recurrent or persistent painless swelling of one or both lips. It may occur without other signs of disease, but it is also a manifestation of Melkersson-Rosenthal syndrome and it may be a presenting symptom of Crohn disease or, rarely, sarcoidosis.2 The term “orofacial granulomatosis” was introduced to encompass the broad spectrum of nonnecrotizing granulomatous inflammation in the orofacial region, including cheilitis granulomatosa, the complete Melkersson-Rosenthal syndrome, sarcoidosis, Crohn disease, and infectious disorders such as tuberculosis.1

The cause of cheilitis granulomatosa is unknown. Specific T-cell clonality has been identified in several patients with orofacial granulomatosis, suggesting a delayed hypersensitivity response. Moreover, the HLA haplotypes HLA-A2 and HLA-A11 have been found in 25% of patients with orofacial granulomatosis, suggesting a viral etiology. A genetic predisposition may exist in Melkersson- Rosenthal syndrome: siblings have been affected, and otherwise unaffected relatives may have a fissured tongue (lingua plicata).

Melkersson-Rosenthal syndrome, a rare condition, is characterized by a classic triad of recurrent swelling of the lips or face (or both), fissured tongue, and relapsing peripheral facial nerve paralysis. It is an unusual cause of facial swelling that can be confused with angioedema.3 This syndrome can be ruled out in this patient because he has only one of the three classic signs. Contact antigens are sometimes implicated.

DIFFERENTIAL DIAGNOSIS OF CHEILITIS GRANULOMATOSA

The differential diagnosis of cheilitis granulomatosa is extensive and includes amyloidosis, cheilitis glandularis, sarcoidosis, Crohn disease, actinic cheilitis, neoplasms, and infections, such as tuberculosis, syphilis, and leprosy.1

As many as 11% of patients with Crohn disease may develop mucocutaneous lesions. Oral lesions of Crohn disease include apthae, cobblestoning of the buccal mucosa, swelling of one or both lips (soft or rubbery), vertical clefts of the lips, or hypertrophic gingivitis. Only 5% of patients with Crohn disease ever develop cheilitis granulomatosa, though most cases occur in children.

Ultimately, the diagnosis of cheilitis granulomatosa is made by correlating the patient’s history and clinical features, usually supported by histopathologic findings of nonnecrotic granulomas extending into the deep dermis, composed of histiocytes and giant cells and associated with a lymphomonocytic infiltrate.

TREATMENT

Treatment of cheilitis granulomatosa is difficult because the cause is unknown and the rate of recurrence is high. Response to treatment is often late and unpredictable. Corticosteroids, clofazimine (Lamprene), and surgical intervention such as cheiloplasty have been described as treatment options. Other treatment options include thalidomide (Thalomid), sulfasalazine (Sulfazine), erythromycin, azathioprine (Imuran), and cyclosporine (Sandimmune). Infliximab (Remicade) has been recently reported as a new alternative treatment, in particular for Melkersson-Rosenthal syndrome.4

In our patient, twice-monthly injections of 1 mL of triamcinolone acetonide 10 mg/mL into the affected lip brought acceptable improvement at 3 months. The patient is on maintenance treatment with twice-monthly triamcinolone injections and has had no relapses after 2 years.

References
  1. van der Waal RI, Schulten EA, van de Scheur MR, Wauters IM, Starink TM, van der Waal I. Cheilitis granulomatosa. J Eur Acad Dermatol Venereol 2001; 15:519523.
  2. van der Waal RI, Schulten EA, van der Meij EH, van de Scheur MR, Starink TM, van der Waal I. Cheilitis granulomatosa: overview of 13 patients with long-term follow-up—results of management. Int J Dermatol 2002; 41:225229.
  3. Kakimoto C, Sparks C, White AA. Melkersson-Rosenthal syndrome: a form of pseudoangioedema. Ann Allergy Asthma Immunol 2007; 99:185189.
  4. Ratzinger G, Sepp N, Vogetseder W, Tilg H. Cheilitis granulomatosa and Melkersson-Rosenthal syndrome: evaluation of gastrointestinal involvement and therapeutic regimens in a series of 14 patients. J Eur Acad Dermatol Venereol 2007; 21:10651070.
Article PDF
Author and Disclosure Information

Sergio Vañó-Galván, MD
Department of Dermatology, Ramón y Cajal Hospital, University of Alcalá, Madrid, Spain

Paula Moreno-Martin, MD
Department of Ophthalmology, La Princesa University Hospital, Madrid, Spain

José-María Arrazola, MD
Department of Dermatology, Ramón y Cajal Hospital, University of Alcalá, Madrid, Spain

Pedro Jaén, PhD
Department of Dermatology, Ramón y Cajal Hospital, University of Alcalá, Madrid, Spain

Address: Sergio Vañó-Galván, MD, Department of Dermatology, Ramón y Cajal Hospital, University of Alcalá, Carretera de Colmenar Viejo, km 9.100, 28034 Madrid, Spain; e-mail sergiovano@yahoo.es

Issue
Cleveland Clinic Journal of Medicine - 76(1)
Publications
Topics
Page Number
12, 15
Sections
Author and Disclosure Information

Sergio Vañó-Galván, MD
Department of Dermatology, Ramón y Cajal Hospital, University of Alcalá, Madrid, Spain

Paula Moreno-Martin, MD
Department of Ophthalmology, La Princesa University Hospital, Madrid, Spain

José-María Arrazola, MD
Department of Dermatology, Ramón y Cajal Hospital, University of Alcalá, Madrid, Spain

Pedro Jaén, PhD
Department of Dermatology, Ramón y Cajal Hospital, University of Alcalá, Madrid, Spain

Address: Sergio Vañó-Galván, MD, Department of Dermatology, Ramón y Cajal Hospital, University of Alcalá, Carretera de Colmenar Viejo, km 9.100, 28034 Madrid, Spain; e-mail sergiovano@yahoo.es

Author and Disclosure Information

Sergio Vañó-Galván, MD
Department of Dermatology, Ramón y Cajal Hospital, University of Alcalá, Madrid, Spain

Paula Moreno-Martin, MD
Department of Ophthalmology, La Princesa University Hospital, Madrid, Spain

José-María Arrazola, MD
Department of Dermatology, Ramón y Cajal Hospital, University of Alcalá, Madrid, Spain

Pedro Jaén, PhD
Department of Dermatology, Ramón y Cajal Hospital, University of Alcalá, Madrid, Spain

Address: Sergio Vañó-Galván, MD, Department of Dermatology, Ramón y Cajal Hospital, University of Alcalá, Carretera de Colmenar Viejo, km 9.100, 28034 Madrid, Spain; e-mail sergiovano@yahoo.es

Article PDF
Article PDF

A 44-year-old man is referred for evaluation of asymptomatic swelling of the lower lip that has persisted for 10 months. He has been treated unsuccessfully with oral antihistamines for suspected chronic angioedema. He has no other symptoms and appears to be well otherwise. He has no history of applied irritants or local trauma, and his medical history is unremarkable.

Figure 1.
Physical examination shows a swollen and indurated lower lip (Figure 1). The tongue and gums are normal, and the ophthalmologic evaluation reveals no abnormalities. No facial paralysis is observed.

Results of the laboratory evaluation, including serum angiotensin-converting enzyme level, are normal. Patch tests to detect contact sensitivity to food additives are negative. Biopsy of the affected lip reveals dense infiltrate of the submucosal connective tissue with focal nonnecrotizing granulomas. Imaging and endoscopic studies show no evidence of sarcoidosis or Crohn disease.

Q: Given what we know so far, which of the following is the most likely diagnosis of the persistent lip swelling?

  • Melkersson-Rosenthal syndrome
  • Amyloidosis
  • Quincke edema
  • Cheilitis granulomatosa
  • Cutaneous tuberculosis

A: From what we know so far, the correct answer is cheilitis granulomatosa. While this rare condition may be a feature of Melkersson-Rosenthal syndrome and amyloidosis, at this point in the evaluation these have not been confirmed. Quincke edema (ie, angioedema) is unlikely, given the ineffectiveness of previous treatment with oral antihistamines. Cutaneous tuberculosis usually presents as “lupus vulgaris,” which is characterized by solitary, small, sharply marginated, red-brown papules of gelatinous consistency (“apple-jelly nodules”), mainly on the head and neck.

Cheilitis granulomatosa is a rare inflammatory disorder1 that primarily affects young adults. Its key feature is recurrent or persistent painless swelling of one or both lips. It may occur without other signs of disease, but it is also a manifestation of Melkersson-Rosenthal syndrome and it may be a presenting symptom of Crohn disease or, rarely, sarcoidosis.2 The term “orofacial granulomatosis” was introduced to encompass the broad spectrum of nonnecrotizing granulomatous inflammation in the orofacial region, including cheilitis granulomatosa, the complete Melkersson-Rosenthal syndrome, sarcoidosis, Crohn disease, and infectious disorders such as tuberculosis.1

The cause of cheilitis granulomatosa is unknown. Specific T-cell clonality has been identified in several patients with orofacial granulomatosis, suggesting a delayed hypersensitivity response. Moreover, the HLA haplotypes HLA-A2 and HLA-A11 have been found in 25% of patients with orofacial granulomatosis, suggesting a viral etiology. A genetic predisposition may exist in Melkersson- Rosenthal syndrome: siblings have been affected, and otherwise unaffected relatives may have a fissured tongue (lingua plicata).

Melkersson-Rosenthal syndrome, a rare condition, is characterized by a classic triad of recurrent swelling of the lips or face (or both), fissured tongue, and relapsing peripheral facial nerve paralysis. It is an unusual cause of facial swelling that can be confused with angioedema.3 This syndrome can be ruled out in this patient because he has only one of the three classic signs. Contact antigens are sometimes implicated.

DIFFERENTIAL DIAGNOSIS OF CHEILITIS GRANULOMATOSA

The differential diagnosis of cheilitis granulomatosa is extensive and includes amyloidosis, cheilitis glandularis, sarcoidosis, Crohn disease, actinic cheilitis, neoplasms, and infections, such as tuberculosis, syphilis, and leprosy.1

As many as 11% of patients with Crohn disease may develop mucocutaneous lesions. Oral lesions of Crohn disease include apthae, cobblestoning of the buccal mucosa, swelling of one or both lips (soft or rubbery), vertical clefts of the lips, or hypertrophic gingivitis. Only 5% of patients with Crohn disease ever develop cheilitis granulomatosa, though most cases occur in children.

Ultimately, the diagnosis of cheilitis granulomatosa is made by correlating the patient’s history and clinical features, usually supported by histopathologic findings of nonnecrotic granulomas extending into the deep dermis, composed of histiocytes and giant cells and associated with a lymphomonocytic infiltrate.

TREATMENT

Treatment of cheilitis granulomatosa is difficult because the cause is unknown and the rate of recurrence is high. Response to treatment is often late and unpredictable. Corticosteroids, clofazimine (Lamprene), and surgical intervention such as cheiloplasty have been described as treatment options. Other treatment options include thalidomide (Thalomid), sulfasalazine (Sulfazine), erythromycin, azathioprine (Imuran), and cyclosporine (Sandimmune). Infliximab (Remicade) has been recently reported as a new alternative treatment, in particular for Melkersson-Rosenthal syndrome.4

In our patient, twice-monthly injections of 1 mL of triamcinolone acetonide 10 mg/mL into the affected lip brought acceptable improvement at 3 months. The patient is on maintenance treatment with twice-monthly triamcinolone injections and has had no relapses after 2 years.

A 44-year-old man is referred for evaluation of asymptomatic swelling of the lower lip that has persisted for 10 months. He has been treated unsuccessfully with oral antihistamines for suspected chronic angioedema. He has no other symptoms and appears to be well otherwise. He has no history of applied irritants or local trauma, and his medical history is unremarkable.

Figure 1.
Physical examination shows a swollen and indurated lower lip (Figure 1). The tongue and gums are normal, and the ophthalmologic evaluation reveals no abnormalities. No facial paralysis is observed.

Results of the laboratory evaluation, including serum angiotensin-converting enzyme level, are normal. Patch tests to detect contact sensitivity to food additives are negative. Biopsy of the affected lip reveals dense infiltrate of the submucosal connective tissue with focal nonnecrotizing granulomas. Imaging and endoscopic studies show no evidence of sarcoidosis or Crohn disease.

Q: Given what we know so far, which of the following is the most likely diagnosis of the persistent lip swelling?

  • Melkersson-Rosenthal syndrome
  • Amyloidosis
  • Quincke edema
  • Cheilitis granulomatosa
  • Cutaneous tuberculosis

A: From what we know so far, the correct answer is cheilitis granulomatosa. While this rare condition may be a feature of Melkersson-Rosenthal syndrome and amyloidosis, at this point in the evaluation these have not been confirmed. Quincke edema (ie, angioedema) is unlikely, given the ineffectiveness of previous treatment with oral antihistamines. Cutaneous tuberculosis usually presents as “lupus vulgaris,” which is characterized by solitary, small, sharply marginated, red-brown papules of gelatinous consistency (“apple-jelly nodules”), mainly on the head and neck.

Cheilitis granulomatosa is a rare inflammatory disorder1 that primarily affects young adults. Its key feature is recurrent or persistent painless swelling of one or both lips. It may occur without other signs of disease, but it is also a manifestation of Melkersson-Rosenthal syndrome and it may be a presenting symptom of Crohn disease or, rarely, sarcoidosis.2 The term “orofacial granulomatosis” was introduced to encompass the broad spectrum of nonnecrotizing granulomatous inflammation in the orofacial region, including cheilitis granulomatosa, the complete Melkersson-Rosenthal syndrome, sarcoidosis, Crohn disease, and infectious disorders such as tuberculosis.1

The cause of cheilitis granulomatosa is unknown. Specific T-cell clonality has been identified in several patients with orofacial granulomatosis, suggesting a delayed hypersensitivity response. Moreover, the HLA haplotypes HLA-A2 and HLA-A11 have been found in 25% of patients with orofacial granulomatosis, suggesting a viral etiology. A genetic predisposition may exist in Melkersson- Rosenthal syndrome: siblings have been affected, and otherwise unaffected relatives may have a fissured tongue (lingua plicata).

Melkersson-Rosenthal syndrome, a rare condition, is characterized by a classic triad of recurrent swelling of the lips or face (or both), fissured tongue, and relapsing peripheral facial nerve paralysis. It is an unusual cause of facial swelling that can be confused with angioedema.3 This syndrome can be ruled out in this patient because he has only one of the three classic signs. Contact antigens are sometimes implicated.

DIFFERENTIAL DIAGNOSIS OF CHEILITIS GRANULOMATOSA

The differential diagnosis of cheilitis granulomatosa is extensive and includes amyloidosis, cheilitis glandularis, sarcoidosis, Crohn disease, actinic cheilitis, neoplasms, and infections, such as tuberculosis, syphilis, and leprosy.1

As many as 11% of patients with Crohn disease may develop mucocutaneous lesions. Oral lesions of Crohn disease include apthae, cobblestoning of the buccal mucosa, swelling of one or both lips (soft or rubbery), vertical clefts of the lips, or hypertrophic gingivitis. Only 5% of patients with Crohn disease ever develop cheilitis granulomatosa, though most cases occur in children.

Ultimately, the diagnosis of cheilitis granulomatosa is made by correlating the patient’s history and clinical features, usually supported by histopathologic findings of nonnecrotic granulomas extending into the deep dermis, composed of histiocytes and giant cells and associated with a lymphomonocytic infiltrate.

TREATMENT

Treatment of cheilitis granulomatosa is difficult because the cause is unknown and the rate of recurrence is high. Response to treatment is often late and unpredictable. Corticosteroids, clofazimine (Lamprene), and surgical intervention such as cheiloplasty have been described as treatment options. Other treatment options include thalidomide (Thalomid), sulfasalazine (Sulfazine), erythromycin, azathioprine (Imuran), and cyclosporine (Sandimmune). Infliximab (Remicade) has been recently reported as a new alternative treatment, in particular for Melkersson-Rosenthal syndrome.4

In our patient, twice-monthly injections of 1 mL of triamcinolone acetonide 10 mg/mL into the affected lip brought acceptable improvement at 3 months. The patient is on maintenance treatment with twice-monthly triamcinolone injections and has had no relapses after 2 years.

References
  1. van der Waal RI, Schulten EA, van de Scheur MR, Wauters IM, Starink TM, van der Waal I. Cheilitis granulomatosa. J Eur Acad Dermatol Venereol 2001; 15:519523.
  2. van der Waal RI, Schulten EA, van der Meij EH, van de Scheur MR, Starink TM, van der Waal I. Cheilitis granulomatosa: overview of 13 patients with long-term follow-up—results of management. Int J Dermatol 2002; 41:225229.
  3. Kakimoto C, Sparks C, White AA. Melkersson-Rosenthal syndrome: a form of pseudoangioedema. Ann Allergy Asthma Immunol 2007; 99:185189.
  4. Ratzinger G, Sepp N, Vogetseder W, Tilg H. Cheilitis granulomatosa and Melkersson-Rosenthal syndrome: evaluation of gastrointestinal involvement and therapeutic regimens in a series of 14 patients. J Eur Acad Dermatol Venereol 2007; 21:10651070.
References
  1. van der Waal RI, Schulten EA, van de Scheur MR, Wauters IM, Starink TM, van der Waal I. Cheilitis granulomatosa. J Eur Acad Dermatol Venereol 2001; 15:519523.
  2. van der Waal RI, Schulten EA, van der Meij EH, van de Scheur MR, Starink TM, van der Waal I. Cheilitis granulomatosa: overview of 13 patients with long-term follow-up—results of management. Int J Dermatol 2002; 41:225229.
  3. Kakimoto C, Sparks C, White AA. Melkersson-Rosenthal syndrome: a form of pseudoangioedema. Ann Allergy Asthma Immunol 2007; 99:185189.
  4. Ratzinger G, Sepp N, Vogetseder W, Tilg H. Cheilitis granulomatosa and Melkersson-Rosenthal syndrome: evaluation of gastrointestinal involvement and therapeutic regimens in a series of 14 patients. J Eur Acad Dermatol Venereol 2007; 21:10651070.
Issue
Cleveland Clinic Journal of Medicine - 76(1)
Issue
Cleveland Clinic Journal of Medicine - 76(1)
Page Number
12, 15
Page Number
12, 15
Publications
Publications
Topics
Article Type
Display Headline
A persistently swollen lip
Display Headline
A persistently swollen lip
Sections
Disallow All Ads
Alternative CME
Article PDF Media

Black hairy tongue

Article Type
Changed
Mon, 04/30/2018 - 12:12
Display Headline
Black hairy tongue

A 71-year-old man presents for evaluation of an asymptomatic black discoloration of the tongue that he noticed several days earlier. The tongue does not itch or hurt, and the patient is otherwise well, although he is concerned about potential malignancy.

He has a history of hypertension, hyperuricemia, and type 2 diabetes treated with oral glucose-lowering drugs, and he has had no recent changes in his medications. He drinks coffee and uses tobacco. His oral hygiene is poor, with intense halitosis.

Figure 1. Elongation of the filiform papillae with a blackish discoloration on the dorsal surface of the tongue.
Physical examination shows a black coloration of the tongue that appears as an elongation of the filiform papillae on the dorsal surface, with no other abnormalities (Figure 1). The physical examination is otherwise normal. Culture of the dorsal surface of the tongue shows no bacterial or fungal overgrowth.

Q: What is the most likely diagnosis?

  • Oral leukoplakia
  • Epidermoid carcinoma of the tongue
  • Malignant melanoma of the tongue
  • Mucosal candidiasis
  • Black hairy tongue

A: Black hairy tongue is correct. A simple treatment consisting of brushing the tongue daily with a soft toothbrush enhanced by previous application of 30% urea is recommended to the patient, and the discoloration resolves completely within 4 weeks. He is educated on correct oral hygiene and discontinues smoking, with no clinical relapses after 2 years of follow-up.

THE CAUSES AND THE COURSE

Black hairy tongue, also known as lingua villosa nigra, is a painless, benign disorder caused by defective desquamation and reactive hypertrophy of the filiform papillae of the tongue. The hairy appearance is due to elongation of keratinized filiform papillae, which may have different colors, varying from white to yellowish brown to black depending on extrinsic factors (eg, tobacco, coffee, tea, food) and intrinsic factors (ie, chromogenic organisms in normal flora).1

The exact pathogenesis is unclear. Precipitating factors include poor oral hygiene, use of the antipsychotic drug olanzapine1 (Zyprexa) or a broad-spectrum antibiotic such as erythromycin,2 and therapeutic radiation of the head and the neck. Tobacco use and drinking coffee and tea are also contributory factors. Neurologic conditions such as trigeminal neuropathy may be associated. 3 Manabe et al4 applied a panel of antikeratin probes, showing that defective desquamation of the cells in the central column of filiform papillae resulted in the formation of highly elongated, cornified spines or “hairs”—the hallmark of lingua villosa nigra.

PRESENTATION AND DIAGNOSIS

Black hairy tongue is usually asymptomatic. However, symptoms such as altered (metallic) taste, nausea, or halitosis may be noted. Most patients with hairy tongue drink coffee or tea, often in addition to tobacco use.

The diagnosis is based on filiform papillae that are elongated more than 3 mm on the dorsal surface of the tongue. Cultures may be taken to rule out a superimposed oral candidiasis or other suspected oral infection.

MANAGEMENT

Although frightening to the patient, black hairy tongue is completely harmless. In most cases, treatment does not require drugs. If fungal overgrowth is present, a topical antifungal can be used when the condition is symptomatic.

Empirical approaches such as brushing or scraping the tongue, improving oral hygiene, and eliminating potential offending factors (eg, tobacco, candies, strong mouthwashes, antibiotics) is usually sufficient to resolve the lesions.5

In our experience, educating the patient about proper oral hygiene (including discontinuing smoking) and encouraging routine tongue brushing are the best preventive and therapeutic measures.

References
  1. Tamam L, Annagur BB. Black hairy tongue associated with olanzapine treatment: a case report. Mt Sinai J Med 2006; 73:891894.
  2. Pigatto PD, Spadari F, Meroni L, Guzzi G. Black hairy tongue associated with long-term oral erythromycin use. J Eur Acad Dermatol Venereol 2008; 22:12691270.
  3. Chesire WP. Unilateral black hairy tongue in trigeminal neuralgia. Headache 2004; 44:908910.
  4. Manabe M, Lim HW, Winzer M, Loomis CA. Architectural organization of filiform papillae in normal and black hairy tongue epithelium: dissection of differentiation pathways in a complex human epithelium according to their patterns of keratin expression. Arch Dermatol 1999; 135:177181.
  5. Sarti GM, Haddy RI, Schaffer D, Kihm J. Black hairy tongue. Am Fam Physician 1990; 41:17511755.
Article PDF
Author and Disclosure Information

Sergio Vañó-Galván, MD
Department of Dermatology, Ramón y Cajal Hospital, University of Alcalá, Madrid, Spain

Pedro Jaén, PhD
Department of Dermatology, Ramón y Cajal Hospital, University of Alcalá, Madrid, Spain

Address: Sergio Vañó-Galván, MD, Department of Dermatology, Ramón y Cajal Hospital, University of Alcalá, Carretera de Colmenar Viejo, km 9.100, 28034 Madrid, Spain; e-mail: sergiovano@yahoo.es

Issue
Cleveland Clinic Journal of Medicine - 75(12)
Publications
Topics
Page Number
847-848
Sections
Author and Disclosure Information

Sergio Vañó-Galván, MD
Department of Dermatology, Ramón y Cajal Hospital, University of Alcalá, Madrid, Spain

Pedro Jaén, PhD
Department of Dermatology, Ramón y Cajal Hospital, University of Alcalá, Madrid, Spain

Address: Sergio Vañó-Galván, MD, Department of Dermatology, Ramón y Cajal Hospital, University of Alcalá, Carretera de Colmenar Viejo, km 9.100, 28034 Madrid, Spain; e-mail: sergiovano@yahoo.es

Author and Disclosure Information

Sergio Vañó-Galván, MD
Department of Dermatology, Ramón y Cajal Hospital, University of Alcalá, Madrid, Spain

Pedro Jaén, PhD
Department of Dermatology, Ramón y Cajal Hospital, University of Alcalá, Madrid, Spain

Address: Sergio Vañó-Galván, MD, Department of Dermatology, Ramón y Cajal Hospital, University of Alcalá, Carretera de Colmenar Viejo, km 9.100, 28034 Madrid, Spain; e-mail: sergiovano@yahoo.es

Article PDF
Article PDF

A 71-year-old man presents for evaluation of an asymptomatic black discoloration of the tongue that he noticed several days earlier. The tongue does not itch or hurt, and the patient is otherwise well, although he is concerned about potential malignancy.

He has a history of hypertension, hyperuricemia, and type 2 diabetes treated with oral glucose-lowering drugs, and he has had no recent changes in his medications. He drinks coffee and uses tobacco. His oral hygiene is poor, with intense halitosis.

Figure 1. Elongation of the filiform papillae with a blackish discoloration on the dorsal surface of the tongue.
Physical examination shows a black coloration of the tongue that appears as an elongation of the filiform papillae on the dorsal surface, with no other abnormalities (Figure 1). The physical examination is otherwise normal. Culture of the dorsal surface of the tongue shows no bacterial or fungal overgrowth.

Q: What is the most likely diagnosis?

  • Oral leukoplakia
  • Epidermoid carcinoma of the tongue
  • Malignant melanoma of the tongue
  • Mucosal candidiasis
  • Black hairy tongue

A: Black hairy tongue is correct. A simple treatment consisting of brushing the tongue daily with a soft toothbrush enhanced by previous application of 30% urea is recommended to the patient, and the discoloration resolves completely within 4 weeks. He is educated on correct oral hygiene and discontinues smoking, with no clinical relapses after 2 years of follow-up.

THE CAUSES AND THE COURSE

Black hairy tongue, also known as lingua villosa nigra, is a painless, benign disorder caused by defective desquamation and reactive hypertrophy of the filiform papillae of the tongue. The hairy appearance is due to elongation of keratinized filiform papillae, which may have different colors, varying from white to yellowish brown to black depending on extrinsic factors (eg, tobacco, coffee, tea, food) and intrinsic factors (ie, chromogenic organisms in normal flora).1

The exact pathogenesis is unclear. Precipitating factors include poor oral hygiene, use of the antipsychotic drug olanzapine1 (Zyprexa) or a broad-spectrum antibiotic such as erythromycin,2 and therapeutic radiation of the head and the neck. Tobacco use and drinking coffee and tea are also contributory factors. Neurologic conditions such as trigeminal neuropathy may be associated. 3 Manabe et al4 applied a panel of antikeratin probes, showing that defective desquamation of the cells in the central column of filiform papillae resulted in the formation of highly elongated, cornified spines or “hairs”—the hallmark of lingua villosa nigra.

PRESENTATION AND DIAGNOSIS

Black hairy tongue is usually asymptomatic. However, symptoms such as altered (metallic) taste, nausea, or halitosis may be noted. Most patients with hairy tongue drink coffee or tea, often in addition to tobacco use.

The diagnosis is based on filiform papillae that are elongated more than 3 mm on the dorsal surface of the tongue. Cultures may be taken to rule out a superimposed oral candidiasis or other suspected oral infection.

MANAGEMENT

Although frightening to the patient, black hairy tongue is completely harmless. In most cases, treatment does not require drugs. If fungal overgrowth is present, a topical antifungal can be used when the condition is symptomatic.

Empirical approaches such as brushing or scraping the tongue, improving oral hygiene, and eliminating potential offending factors (eg, tobacco, candies, strong mouthwashes, antibiotics) is usually sufficient to resolve the lesions.5

In our experience, educating the patient about proper oral hygiene (including discontinuing smoking) and encouraging routine tongue brushing are the best preventive and therapeutic measures.

A 71-year-old man presents for evaluation of an asymptomatic black discoloration of the tongue that he noticed several days earlier. The tongue does not itch or hurt, and the patient is otherwise well, although he is concerned about potential malignancy.

He has a history of hypertension, hyperuricemia, and type 2 diabetes treated with oral glucose-lowering drugs, and he has had no recent changes in his medications. He drinks coffee and uses tobacco. His oral hygiene is poor, with intense halitosis.

Figure 1. Elongation of the filiform papillae with a blackish discoloration on the dorsal surface of the tongue.
Physical examination shows a black coloration of the tongue that appears as an elongation of the filiform papillae on the dorsal surface, with no other abnormalities (Figure 1). The physical examination is otherwise normal. Culture of the dorsal surface of the tongue shows no bacterial or fungal overgrowth.

Q: What is the most likely diagnosis?

  • Oral leukoplakia
  • Epidermoid carcinoma of the tongue
  • Malignant melanoma of the tongue
  • Mucosal candidiasis
  • Black hairy tongue

A: Black hairy tongue is correct. A simple treatment consisting of brushing the tongue daily with a soft toothbrush enhanced by previous application of 30% urea is recommended to the patient, and the discoloration resolves completely within 4 weeks. He is educated on correct oral hygiene and discontinues smoking, with no clinical relapses after 2 years of follow-up.

THE CAUSES AND THE COURSE

Black hairy tongue, also known as lingua villosa nigra, is a painless, benign disorder caused by defective desquamation and reactive hypertrophy of the filiform papillae of the tongue. The hairy appearance is due to elongation of keratinized filiform papillae, which may have different colors, varying from white to yellowish brown to black depending on extrinsic factors (eg, tobacco, coffee, tea, food) and intrinsic factors (ie, chromogenic organisms in normal flora).1

The exact pathogenesis is unclear. Precipitating factors include poor oral hygiene, use of the antipsychotic drug olanzapine1 (Zyprexa) or a broad-spectrum antibiotic such as erythromycin,2 and therapeutic radiation of the head and the neck. Tobacco use and drinking coffee and tea are also contributory factors. Neurologic conditions such as trigeminal neuropathy may be associated. 3 Manabe et al4 applied a panel of antikeratin probes, showing that defective desquamation of the cells in the central column of filiform papillae resulted in the formation of highly elongated, cornified spines or “hairs”—the hallmark of lingua villosa nigra.

PRESENTATION AND DIAGNOSIS

Black hairy tongue is usually asymptomatic. However, symptoms such as altered (metallic) taste, nausea, or halitosis may be noted. Most patients with hairy tongue drink coffee or tea, often in addition to tobacco use.

The diagnosis is based on filiform papillae that are elongated more than 3 mm on the dorsal surface of the tongue. Cultures may be taken to rule out a superimposed oral candidiasis or other suspected oral infection.

MANAGEMENT

Although frightening to the patient, black hairy tongue is completely harmless. In most cases, treatment does not require drugs. If fungal overgrowth is present, a topical antifungal can be used when the condition is symptomatic.

Empirical approaches such as brushing or scraping the tongue, improving oral hygiene, and eliminating potential offending factors (eg, tobacco, candies, strong mouthwashes, antibiotics) is usually sufficient to resolve the lesions.5

In our experience, educating the patient about proper oral hygiene (including discontinuing smoking) and encouraging routine tongue brushing are the best preventive and therapeutic measures.

References
  1. Tamam L, Annagur BB. Black hairy tongue associated with olanzapine treatment: a case report. Mt Sinai J Med 2006; 73:891894.
  2. Pigatto PD, Spadari F, Meroni L, Guzzi G. Black hairy tongue associated with long-term oral erythromycin use. J Eur Acad Dermatol Venereol 2008; 22:12691270.
  3. Chesire WP. Unilateral black hairy tongue in trigeminal neuralgia. Headache 2004; 44:908910.
  4. Manabe M, Lim HW, Winzer M, Loomis CA. Architectural organization of filiform papillae in normal and black hairy tongue epithelium: dissection of differentiation pathways in a complex human epithelium according to their patterns of keratin expression. Arch Dermatol 1999; 135:177181.
  5. Sarti GM, Haddy RI, Schaffer D, Kihm J. Black hairy tongue. Am Fam Physician 1990; 41:17511755.
References
  1. Tamam L, Annagur BB. Black hairy tongue associated with olanzapine treatment: a case report. Mt Sinai J Med 2006; 73:891894.
  2. Pigatto PD, Spadari F, Meroni L, Guzzi G. Black hairy tongue associated with long-term oral erythromycin use. J Eur Acad Dermatol Venereol 2008; 22:12691270.
  3. Chesire WP. Unilateral black hairy tongue in trigeminal neuralgia. Headache 2004; 44:908910.
  4. Manabe M, Lim HW, Winzer M, Loomis CA. Architectural organization of filiform papillae in normal and black hairy tongue epithelium: dissection of differentiation pathways in a complex human epithelium according to their patterns of keratin expression. Arch Dermatol 1999; 135:177181.
  5. Sarti GM, Haddy RI, Schaffer D, Kihm J. Black hairy tongue. Am Fam Physician 1990; 41:17511755.
Issue
Cleveland Clinic Journal of Medicine - 75(12)
Issue
Cleveland Clinic Journal of Medicine - 75(12)
Page Number
847-848
Page Number
847-848
Publications
Publications
Topics
Article Type
Display Headline
Black hairy tongue
Display Headline
Black hairy tongue
Sections
Disallow All Ads
Alternative CME
Use ProPublica
Article PDF Media

Sudden hair loss associated with trachyonychia

Article Type
Changed
Mon, 04/23/2018 - 11:35
Display Headline
Sudden hair loss associated with trachyonychia

A 30-year-old woman has a 6-month history of episodes of sudden loss of hair in well-demarcated, localized areas of her scalp. She is otherwise well and has had no pruritus or pain. She has had atopic dermatitis since childhood, occasionally treated with topical corticosteroids, and localized facial vitiligo for the past 8 years. She takes no systemic drugs. She has no family history of similar lesions.

Figure 1. Skin-colored alopecic patches on the vertex.
Her head has multiple round to oval skin-colored, well-demarcated alopecic patches on the vertex (Figure 1). Her fingernails and toe-nails have a “sandpapered” appearance, with a rough, lusterless nail plate characteristic of trachyonychia (Figure 2). Otherwise, the physical examination is normal.

Figure 2. Trachyonychia.
Histologic examination of the scalp shows a peribulbar lymphocytic infiltrate similar in appearance to a swarm of bees. Routine laboratory tests, including thyroid hormones and thyroid-stimulating hormone, are normal.

Q: What is the most likely diagnosis?

  • Telogen effluvium
  • Syphilitic alopecia
  • Alopecia areata
  • Trichotillomania
  • Androgenetic alopecia

A: The correct answer is alopecia areata, a disease characterized by recurrent episodes of sudden hair loss and nail changes such as trachyonychia. It can affect any hair-bearing area, but the scalp is the most common site (90% of cases).1 The condition can range from a single patch of hair loss to complete loss of hair on the scalp (alopecia totalis) or the entire body (alopecia universalis).

The characteristic lesion is a round or oval, totally bald, smooth patch, which may have a mild peachy hue. A frequent feature is “exclamation-mark” hairs at the margin of the patch, appearing as broad distal hair shafts, narrow proximally. Active alopecia areata can be confirmed with a positive hair-pull test.

Nail involvement is common in alopecia areata, particularly in severe forms. Pitting is the most common finding, but other abnormalities such as trachyonychia have been reported.1 Nail plate pitting tends to be more shallow and regularly spaced than in psori-atic nail pitting. Trachyonychia, also known as “twenty-nail dystrophy,” is the term used to describe nail plate roughness, pitting, and ridging, affecting 1 to 20 nails. It is associated with a number of skin conditions, including alopecia areata, psoriasis, lichen planus, atopic dermatitis, and ichthyosis vulgaris.4

AN AUTOIMMUNE DISEASE

The precise cause of this common disorder has not been elucidated, but substantial evidence suggests that it is an organ-specific autoimmune disease that targets hair follicles.1–3

The current hypothesis is that the hair follicle is a site of immune privilege. In alopecia areata, increased expression of major histocompatibility complex class I molecules and down-regulation of immunosuppressants allows the immune system to recognize the immune-privileged follicle antigens.2 Several autoimmune diseases, such as thyroid disorders, vitiligo, pernicious anemia, diabetes mellitus, lupus erythematosus, myasthenia gravis, lichen planus, autoimmune polyendocrine syndrome type I, and celiac disease, have been reported in association with alopecia areata1; thyroid disorders and vitiligo have the strongest relationship. Other diseases reported to be associated with alopecia areata at a higher rate than in the general population are atopic dermatitis and Down syndrome.1

NO CURE, BUT TREATABLE

The choice of treatment depends on the patient’s age and the extent of alopecia activity.5 No cure or preventive treatment has been found, so treatments are directed toward halting disease activity.

Corticosteroids are currently the most popular form of treatment, and published reports support their efficacy.1 Topical and intralesional steroids are usually the first-line treatment for localized disease. Systemic steroids are rarely used, except in cases with rapid progression.

Other treatments that have been used with some success include minoxidil (Rogaine), anthralin (Dritho-Scalp), contact sensitizers (dinitrochlorobenzene, diphenylcycloprope-none), PUVA (psoralen plus ultraviolet A light), and cyclosporine (Sandimmune).1 If the alopecia is resistant to therapy, hair pros-theses can be recommended.

Indicators of poor prognosis (ie, hair that will not regrow completely) include atopy, the presence of other immune diseases, young age at onset, family history, nail dystrophy, extensive hair loss, and ophiasis (a continuous band of baldness involving the temporal and occipital margins).

Because current treatments may not bring results for 3 to 6 months, it is essential to reassure and inform the patient about the results that can be expected. Physicians should also inform patients of the chronic relapsing nature of alopecia areata and its unpredictable course. Although the condition does not threaten the patient’s general health and does not cause scarring, its psychological impact is significant. Support groups such as the National Alopecia Areata Foundation (www.naaf.org) can help.

References
  1. Wasserman D, Guzman-Sanchez DA, Scott K, McMichael A. Alopecia areata. Int J Dermatol 2007; 46:121131.
  2. Paus R, Ito N, Takigawa M, Ito T. The hair follicle and immune privilege. J Investig Dermatol Symp Proc 2003; 8:188194.
  3. Alexis AF, Dudda-Subramanya R, Sinha AA. Alopecia areata: autoimmune basis of hair loss. Eur J Dermatol 2004; 14:364370.
  4. Blanco FP, Scher RK. Trachyonychia: case report and review of the literature. J Drugs Dermatol 2006; 5:469472.
  5. Dombrowski NC, Bergfeld WF. Alopecia areata: what to expect from current treatments. Cleve Clin J Med 2005; 72:758768.
Article PDF
Author and Disclosure Information

Sergio Vañó-Galván, MD
Department of Dermatology, Ramón y Cajal Hospital, University of Alcalá, Madrid, Spain

Sonsoles Aboín, MD
Department of Dermatology, Ramón y Cajal Hospital, Madrid, Spain

Sonio Beà-Ardebol, MD
Department of Dermatology, Ramón y Cajal Hospital, Madrid, Spain

Juan Luis Santiago Sánchez-Mateos, MD
Department of Dermatology, Ramón y Cajal University Hospital, Madrid, Spain

Address: Sergio Vañó-Galván, MD, Department of Dermatology, Ramón y Cajal University Hospital, Carretera de Colmenar Viejo, km 9.100, 28034 Madrid, Spain; e-mail sergiovano@yahoo.es

Issue
Cleveland Clinic Journal of Medicine - 75(8)
Publications
Topics
Page Number
567-568
Sections
Author and Disclosure Information

Sergio Vañó-Galván, MD
Department of Dermatology, Ramón y Cajal Hospital, University of Alcalá, Madrid, Spain

Sonsoles Aboín, MD
Department of Dermatology, Ramón y Cajal Hospital, Madrid, Spain

Sonio Beà-Ardebol, MD
Department of Dermatology, Ramón y Cajal Hospital, Madrid, Spain

Juan Luis Santiago Sánchez-Mateos, MD
Department of Dermatology, Ramón y Cajal University Hospital, Madrid, Spain

Address: Sergio Vañó-Galván, MD, Department of Dermatology, Ramón y Cajal University Hospital, Carretera de Colmenar Viejo, km 9.100, 28034 Madrid, Spain; e-mail sergiovano@yahoo.es

Author and Disclosure Information

Sergio Vañó-Galván, MD
Department of Dermatology, Ramón y Cajal Hospital, University of Alcalá, Madrid, Spain

Sonsoles Aboín, MD
Department of Dermatology, Ramón y Cajal Hospital, Madrid, Spain

Sonio Beà-Ardebol, MD
Department of Dermatology, Ramón y Cajal Hospital, Madrid, Spain

Juan Luis Santiago Sánchez-Mateos, MD
Department of Dermatology, Ramón y Cajal University Hospital, Madrid, Spain

Address: Sergio Vañó-Galván, MD, Department of Dermatology, Ramón y Cajal University Hospital, Carretera de Colmenar Viejo, km 9.100, 28034 Madrid, Spain; e-mail sergiovano@yahoo.es

Article PDF
Article PDF

A 30-year-old woman has a 6-month history of episodes of sudden loss of hair in well-demarcated, localized areas of her scalp. She is otherwise well and has had no pruritus or pain. She has had atopic dermatitis since childhood, occasionally treated with topical corticosteroids, and localized facial vitiligo for the past 8 years. She takes no systemic drugs. She has no family history of similar lesions.

Figure 1. Skin-colored alopecic patches on the vertex.
Her head has multiple round to oval skin-colored, well-demarcated alopecic patches on the vertex (Figure 1). Her fingernails and toe-nails have a “sandpapered” appearance, with a rough, lusterless nail plate characteristic of trachyonychia (Figure 2). Otherwise, the physical examination is normal.

Figure 2. Trachyonychia.
Histologic examination of the scalp shows a peribulbar lymphocytic infiltrate similar in appearance to a swarm of bees. Routine laboratory tests, including thyroid hormones and thyroid-stimulating hormone, are normal.

Q: What is the most likely diagnosis?

  • Telogen effluvium
  • Syphilitic alopecia
  • Alopecia areata
  • Trichotillomania
  • Androgenetic alopecia

A: The correct answer is alopecia areata, a disease characterized by recurrent episodes of sudden hair loss and nail changes such as trachyonychia. It can affect any hair-bearing area, but the scalp is the most common site (90% of cases).1 The condition can range from a single patch of hair loss to complete loss of hair on the scalp (alopecia totalis) or the entire body (alopecia universalis).

The characteristic lesion is a round or oval, totally bald, smooth patch, which may have a mild peachy hue. A frequent feature is “exclamation-mark” hairs at the margin of the patch, appearing as broad distal hair shafts, narrow proximally. Active alopecia areata can be confirmed with a positive hair-pull test.

Nail involvement is common in alopecia areata, particularly in severe forms. Pitting is the most common finding, but other abnormalities such as trachyonychia have been reported.1 Nail plate pitting tends to be more shallow and regularly spaced than in psori-atic nail pitting. Trachyonychia, also known as “twenty-nail dystrophy,” is the term used to describe nail plate roughness, pitting, and ridging, affecting 1 to 20 nails. It is associated with a number of skin conditions, including alopecia areata, psoriasis, lichen planus, atopic dermatitis, and ichthyosis vulgaris.4

AN AUTOIMMUNE DISEASE

The precise cause of this common disorder has not been elucidated, but substantial evidence suggests that it is an organ-specific autoimmune disease that targets hair follicles.1–3

The current hypothesis is that the hair follicle is a site of immune privilege. In alopecia areata, increased expression of major histocompatibility complex class I molecules and down-regulation of immunosuppressants allows the immune system to recognize the immune-privileged follicle antigens.2 Several autoimmune diseases, such as thyroid disorders, vitiligo, pernicious anemia, diabetes mellitus, lupus erythematosus, myasthenia gravis, lichen planus, autoimmune polyendocrine syndrome type I, and celiac disease, have been reported in association with alopecia areata1; thyroid disorders and vitiligo have the strongest relationship. Other diseases reported to be associated with alopecia areata at a higher rate than in the general population are atopic dermatitis and Down syndrome.1

NO CURE, BUT TREATABLE

The choice of treatment depends on the patient’s age and the extent of alopecia activity.5 No cure or preventive treatment has been found, so treatments are directed toward halting disease activity.

Corticosteroids are currently the most popular form of treatment, and published reports support their efficacy.1 Topical and intralesional steroids are usually the first-line treatment for localized disease. Systemic steroids are rarely used, except in cases with rapid progression.

Other treatments that have been used with some success include minoxidil (Rogaine), anthralin (Dritho-Scalp), contact sensitizers (dinitrochlorobenzene, diphenylcycloprope-none), PUVA (psoralen plus ultraviolet A light), and cyclosporine (Sandimmune).1 If the alopecia is resistant to therapy, hair pros-theses can be recommended.

Indicators of poor prognosis (ie, hair that will not regrow completely) include atopy, the presence of other immune diseases, young age at onset, family history, nail dystrophy, extensive hair loss, and ophiasis (a continuous band of baldness involving the temporal and occipital margins).

Because current treatments may not bring results for 3 to 6 months, it is essential to reassure and inform the patient about the results that can be expected. Physicians should also inform patients of the chronic relapsing nature of alopecia areata and its unpredictable course. Although the condition does not threaten the patient’s general health and does not cause scarring, its psychological impact is significant. Support groups such as the National Alopecia Areata Foundation (www.naaf.org) can help.

A 30-year-old woman has a 6-month history of episodes of sudden loss of hair in well-demarcated, localized areas of her scalp. She is otherwise well and has had no pruritus or pain. She has had atopic dermatitis since childhood, occasionally treated with topical corticosteroids, and localized facial vitiligo for the past 8 years. She takes no systemic drugs. She has no family history of similar lesions.

Figure 1. Skin-colored alopecic patches on the vertex.
Her head has multiple round to oval skin-colored, well-demarcated alopecic patches on the vertex (Figure 1). Her fingernails and toe-nails have a “sandpapered” appearance, with a rough, lusterless nail plate characteristic of trachyonychia (Figure 2). Otherwise, the physical examination is normal.

Figure 2. Trachyonychia.
Histologic examination of the scalp shows a peribulbar lymphocytic infiltrate similar in appearance to a swarm of bees. Routine laboratory tests, including thyroid hormones and thyroid-stimulating hormone, are normal.

Q: What is the most likely diagnosis?

  • Telogen effluvium
  • Syphilitic alopecia
  • Alopecia areata
  • Trichotillomania
  • Androgenetic alopecia

A: The correct answer is alopecia areata, a disease characterized by recurrent episodes of sudden hair loss and nail changes such as trachyonychia. It can affect any hair-bearing area, but the scalp is the most common site (90% of cases).1 The condition can range from a single patch of hair loss to complete loss of hair on the scalp (alopecia totalis) or the entire body (alopecia universalis).

The characteristic lesion is a round or oval, totally bald, smooth patch, which may have a mild peachy hue. A frequent feature is “exclamation-mark” hairs at the margin of the patch, appearing as broad distal hair shafts, narrow proximally. Active alopecia areata can be confirmed with a positive hair-pull test.

Nail involvement is common in alopecia areata, particularly in severe forms. Pitting is the most common finding, but other abnormalities such as trachyonychia have been reported.1 Nail plate pitting tends to be more shallow and regularly spaced than in psori-atic nail pitting. Trachyonychia, also known as “twenty-nail dystrophy,” is the term used to describe nail plate roughness, pitting, and ridging, affecting 1 to 20 nails. It is associated with a number of skin conditions, including alopecia areata, psoriasis, lichen planus, atopic dermatitis, and ichthyosis vulgaris.4

AN AUTOIMMUNE DISEASE

The precise cause of this common disorder has not been elucidated, but substantial evidence suggests that it is an organ-specific autoimmune disease that targets hair follicles.1–3

The current hypothesis is that the hair follicle is a site of immune privilege. In alopecia areata, increased expression of major histocompatibility complex class I molecules and down-regulation of immunosuppressants allows the immune system to recognize the immune-privileged follicle antigens.2 Several autoimmune diseases, such as thyroid disorders, vitiligo, pernicious anemia, diabetes mellitus, lupus erythematosus, myasthenia gravis, lichen planus, autoimmune polyendocrine syndrome type I, and celiac disease, have been reported in association with alopecia areata1; thyroid disorders and vitiligo have the strongest relationship. Other diseases reported to be associated with alopecia areata at a higher rate than in the general population are atopic dermatitis and Down syndrome.1

NO CURE, BUT TREATABLE

The choice of treatment depends on the patient’s age and the extent of alopecia activity.5 No cure or preventive treatment has been found, so treatments are directed toward halting disease activity.

Corticosteroids are currently the most popular form of treatment, and published reports support their efficacy.1 Topical and intralesional steroids are usually the first-line treatment for localized disease. Systemic steroids are rarely used, except in cases with rapid progression.

Other treatments that have been used with some success include minoxidil (Rogaine), anthralin (Dritho-Scalp), contact sensitizers (dinitrochlorobenzene, diphenylcycloprope-none), PUVA (psoralen plus ultraviolet A light), and cyclosporine (Sandimmune).1 If the alopecia is resistant to therapy, hair pros-theses can be recommended.

Indicators of poor prognosis (ie, hair that will not regrow completely) include atopy, the presence of other immune diseases, young age at onset, family history, nail dystrophy, extensive hair loss, and ophiasis (a continuous band of baldness involving the temporal and occipital margins).

Because current treatments may not bring results for 3 to 6 months, it is essential to reassure and inform the patient about the results that can be expected. Physicians should also inform patients of the chronic relapsing nature of alopecia areata and its unpredictable course. Although the condition does not threaten the patient’s general health and does not cause scarring, its psychological impact is significant. Support groups such as the National Alopecia Areata Foundation (www.naaf.org) can help.

References
  1. Wasserman D, Guzman-Sanchez DA, Scott K, McMichael A. Alopecia areata. Int J Dermatol 2007; 46:121131.
  2. Paus R, Ito N, Takigawa M, Ito T. The hair follicle and immune privilege. J Investig Dermatol Symp Proc 2003; 8:188194.
  3. Alexis AF, Dudda-Subramanya R, Sinha AA. Alopecia areata: autoimmune basis of hair loss. Eur J Dermatol 2004; 14:364370.
  4. Blanco FP, Scher RK. Trachyonychia: case report and review of the literature. J Drugs Dermatol 2006; 5:469472.
  5. Dombrowski NC, Bergfeld WF. Alopecia areata: what to expect from current treatments. Cleve Clin J Med 2005; 72:758768.
References
  1. Wasserman D, Guzman-Sanchez DA, Scott K, McMichael A. Alopecia areata. Int J Dermatol 2007; 46:121131.
  2. Paus R, Ito N, Takigawa M, Ito T. The hair follicle and immune privilege. J Investig Dermatol Symp Proc 2003; 8:188194.
  3. Alexis AF, Dudda-Subramanya R, Sinha AA. Alopecia areata: autoimmune basis of hair loss. Eur J Dermatol 2004; 14:364370.
  4. Blanco FP, Scher RK. Trachyonychia: case report and review of the literature. J Drugs Dermatol 2006; 5:469472.
  5. Dombrowski NC, Bergfeld WF. Alopecia areata: what to expect from current treatments. Cleve Clin J Med 2005; 72:758768.
Issue
Cleveland Clinic Journal of Medicine - 75(8)
Issue
Cleveland Clinic Journal of Medicine - 75(8)
Page Number
567-568
Page Number
567-568
Publications
Publications
Topics
Article Type
Display Headline
Sudden hair loss associated with trachyonychia
Display Headline
Sudden hair loss associated with trachyonychia
Sections
Disallow All Ads
Alternative CME
Use ProPublica
Article PDF Media

Generalized pruritus after a beach vacation

Article Type
Changed
Mon, 04/23/2018 - 09:00
Display Headline
Generalized pruritus after a beach vacation

A 25-year-old man presents with a 2-month history of generalized itching that began 5 weeks after returning from a trip to a beach in Brazil, during which he had sexual relations without protection. He has already been treated with topical steroids and antihistamines, with no improvement.

Figure 1.
On physical examination, he has papules and excoriations on the abdomen (Figure 1A) and buttocks (Figure 1B), a linear grayish threadlike elevation on the flexor surface of the left wrist (Figure 1C), and multiple red nodules on the penis and scrotum (Figure 1D).

Q: What is the most likely diagnosis?

  • Herpes simplex type 2
  • Acute trypanosomiasis
  • Syphilis
  • Scabies
  • Papular urticaria

A: The most likely diagnosis is scabies, an intensely pruritic skin infestation caused by the host-specific mite Sarcoptes scabiei var hominis.1 Herpes simplex type 2 infection presents with clustered vesicles, not nodules, on an erythematous base in the genital area, without generalized pruritus. Acute trypanosomiasis involves malaise, fever, vomiting, diarrhea, anorexia, rash, tachycardia, and even generalized lymphadenopathy and meningeal irritation, but generalized itching is not typical. The location and morphology of the patient’s lesions are not consistent with syphilis. Topical steroids and oral antihistamines should have improved the pruritus of papular urticaria.

A GLOBAL PUBLIC HEALTH PROBLEM

Scabies is a worldwide public health problem, affecting people of all ages, races, and socioeconomic groups.1 Overcrowding, delayed diagnosis and treatment, and poor public education contribute to the prevalence of scabies in both industrialized and nonindustrialized nations.2 Prevalence rates are higher in children and people who are sexually active.3

Sexual transmission is by close skin-to-skin contact. Poor sensory perception in conditions such as leprosy and compromised immunity due to organ transplantation, human immunodeficiency virus infection, or old age increase the risk for the crusted variant of scabies.2 Patients with the crusted variant tend to present with clinically atypical lesions, and because of this they are often misdiag-nosed, thus delaying treatment and elevating the risk of local epidemics.

CLINICAL ASPECTS

Scabies can mimic a broad range of skin diseases. Patients present with intense itching that is worse at night. The face and neck are rarely affected. The pathognomonic signs of scabies are burrows, erythematous papules, and generalized pruritus (also on non-infested skin) with nocturnal predominance.3 Reddish to brownish extremely pruritic nodules of 2 to 20 mm in diameter may be also present on the genitalia (more commonly in males than in females), buttocks, groin, and axillary regions. Patients usually have secondary papules, pustules, vesicles, and excoriations.

DIAGNOSIS

Every patient with intense pruritus should be suspected of having scabies, but especially if a family member reports similar symptoms.3 A diagnosis can be made clinically if a burrow is detected at a typical predilection site and if the lesion itches severely. In this case, even a single burrow is pathognomonic.2 The diagnosis is confirmed by light-microscopic identification of mites, larvae, ova, or scybala (fecal pellets) in skin scrapings.1

TREATMENT

Treatment includes a scabicidal agent, an antipruritic agent such as a sedating antihistamine, and an appropriate antimicrobial agent in cases of secondary infection. Permethrin (Acticin), a 5% synthetic pyrethroid cream, is an excellent scabicide and is the preferred treatment.1 All family members and close contacts must be evaluated and treated, even if they do not have symptoms.1

References
  1. Chosidow O. Clinical practices. Scabies. N Engl J Med 2006; 354:17181727.
  2. Heukelbach J, Feldmeier H. Scabies. Lancet 2006; 367:17671774.
  3. Hengge UR, Currie BJ, Jäger G, Lupi O, Schwartz RA. Scabies: a ubiquitous neglected skin disease. Lancet Infect Dis 2006; 6:769779.
Article PDF
Author and Disclosure Information

Sergio Vañó-Galván, MD
Department of Dermatology, Ramón y Cajal Hospital, University of Alcalá, Madrid, Spain

Paula Moreno-Martin, MD
La Princesa University Hospital, Madrid, Spain

Address: Sergio Vañó-Galván, MD, Department of Dermatology, Ramón y Cajal University Hospital, Carretera de Colmenar Viejo km 9.100, 28034 Madrid, Spain; e-mail sergiovano@yahoo.es.

Issue
Cleveland Clinic Journal of Medicine - 75(7)
Publications
Topics
Page Number
474, 478
Sections
Author and Disclosure Information

Sergio Vañó-Galván, MD
Department of Dermatology, Ramón y Cajal Hospital, University of Alcalá, Madrid, Spain

Paula Moreno-Martin, MD
La Princesa University Hospital, Madrid, Spain

Address: Sergio Vañó-Galván, MD, Department of Dermatology, Ramón y Cajal University Hospital, Carretera de Colmenar Viejo km 9.100, 28034 Madrid, Spain; e-mail sergiovano@yahoo.es.

Author and Disclosure Information

Sergio Vañó-Galván, MD
Department of Dermatology, Ramón y Cajal Hospital, University of Alcalá, Madrid, Spain

Paula Moreno-Martin, MD
La Princesa University Hospital, Madrid, Spain

Address: Sergio Vañó-Galván, MD, Department of Dermatology, Ramón y Cajal University Hospital, Carretera de Colmenar Viejo km 9.100, 28034 Madrid, Spain; e-mail sergiovano@yahoo.es.

Article PDF
Article PDF

A 25-year-old man presents with a 2-month history of generalized itching that began 5 weeks after returning from a trip to a beach in Brazil, during which he had sexual relations without protection. He has already been treated with topical steroids and antihistamines, with no improvement.

Figure 1.
On physical examination, he has papules and excoriations on the abdomen (Figure 1A) and buttocks (Figure 1B), a linear grayish threadlike elevation on the flexor surface of the left wrist (Figure 1C), and multiple red nodules on the penis and scrotum (Figure 1D).

Q: What is the most likely diagnosis?

  • Herpes simplex type 2
  • Acute trypanosomiasis
  • Syphilis
  • Scabies
  • Papular urticaria

A: The most likely diagnosis is scabies, an intensely pruritic skin infestation caused by the host-specific mite Sarcoptes scabiei var hominis.1 Herpes simplex type 2 infection presents with clustered vesicles, not nodules, on an erythematous base in the genital area, without generalized pruritus. Acute trypanosomiasis involves malaise, fever, vomiting, diarrhea, anorexia, rash, tachycardia, and even generalized lymphadenopathy and meningeal irritation, but generalized itching is not typical. The location and morphology of the patient’s lesions are not consistent with syphilis. Topical steroids and oral antihistamines should have improved the pruritus of papular urticaria.

A GLOBAL PUBLIC HEALTH PROBLEM

Scabies is a worldwide public health problem, affecting people of all ages, races, and socioeconomic groups.1 Overcrowding, delayed diagnosis and treatment, and poor public education contribute to the prevalence of scabies in both industrialized and nonindustrialized nations.2 Prevalence rates are higher in children and people who are sexually active.3

Sexual transmission is by close skin-to-skin contact. Poor sensory perception in conditions such as leprosy and compromised immunity due to organ transplantation, human immunodeficiency virus infection, or old age increase the risk for the crusted variant of scabies.2 Patients with the crusted variant tend to present with clinically atypical lesions, and because of this they are often misdiag-nosed, thus delaying treatment and elevating the risk of local epidemics.

CLINICAL ASPECTS

Scabies can mimic a broad range of skin diseases. Patients present with intense itching that is worse at night. The face and neck are rarely affected. The pathognomonic signs of scabies are burrows, erythematous papules, and generalized pruritus (also on non-infested skin) with nocturnal predominance.3 Reddish to brownish extremely pruritic nodules of 2 to 20 mm in diameter may be also present on the genitalia (more commonly in males than in females), buttocks, groin, and axillary regions. Patients usually have secondary papules, pustules, vesicles, and excoriations.

DIAGNOSIS

Every patient with intense pruritus should be suspected of having scabies, but especially if a family member reports similar symptoms.3 A diagnosis can be made clinically if a burrow is detected at a typical predilection site and if the lesion itches severely. In this case, even a single burrow is pathognomonic.2 The diagnosis is confirmed by light-microscopic identification of mites, larvae, ova, or scybala (fecal pellets) in skin scrapings.1

TREATMENT

Treatment includes a scabicidal agent, an antipruritic agent such as a sedating antihistamine, and an appropriate antimicrobial agent in cases of secondary infection. Permethrin (Acticin), a 5% synthetic pyrethroid cream, is an excellent scabicide and is the preferred treatment.1 All family members and close contacts must be evaluated and treated, even if they do not have symptoms.1

A 25-year-old man presents with a 2-month history of generalized itching that began 5 weeks after returning from a trip to a beach in Brazil, during which he had sexual relations without protection. He has already been treated with topical steroids and antihistamines, with no improvement.

Figure 1.
On physical examination, he has papules and excoriations on the abdomen (Figure 1A) and buttocks (Figure 1B), a linear grayish threadlike elevation on the flexor surface of the left wrist (Figure 1C), and multiple red nodules on the penis and scrotum (Figure 1D).

Q: What is the most likely diagnosis?

  • Herpes simplex type 2
  • Acute trypanosomiasis
  • Syphilis
  • Scabies
  • Papular urticaria

A: The most likely diagnosis is scabies, an intensely pruritic skin infestation caused by the host-specific mite Sarcoptes scabiei var hominis.1 Herpes simplex type 2 infection presents with clustered vesicles, not nodules, on an erythematous base in the genital area, without generalized pruritus. Acute trypanosomiasis involves malaise, fever, vomiting, diarrhea, anorexia, rash, tachycardia, and even generalized lymphadenopathy and meningeal irritation, but generalized itching is not typical. The location and morphology of the patient’s lesions are not consistent with syphilis. Topical steroids and oral antihistamines should have improved the pruritus of papular urticaria.

A GLOBAL PUBLIC HEALTH PROBLEM

Scabies is a worldwide public health problem, affecting people of all ages, races, and socioeconomic groups.1 Overcrowding, delayed diagnosis and treatment, and poor public education contribute to the prevalence of scabies in both industrialized and nonindustrialized nations.2 Prevalence rates are higher in children and people who are sexually active.3

Sexual transmission is by close skin-to-skin contact. Poor sensory perception in conditions such as leprosy and compromised immunity due to organ transplantation, human immunodeficiency virus infection, or old age increase the risk for the crusted variant of scabies.2 Patients with the crusted variant tend to present with clinically atypical lesions, and because of this they are often misdiag-nosed, thus delaying treatment and elevating the risk of local epidemics.

CLINICAL ASPECTS

Scabies can mimic a broad range of skin diseases. Patients present with intense itching that is worse at night. The face and neck are rarely affected. The pathognomonic signs of scabies are burrows, erythematous papules, and generalized pruritus (also on non-infested skin) with nocturnal predominance.3 Reddish to brownish extremely pruritic nodules of 2 to 20 mm in diameter may be also present on the genitalia (more commonly in males than in females), buttocks, groin, and axillary regions. Patients usually have secondary papules, pustules, vesicles, and excoriations.

DIAGNOSIS

Every patient with intense pruritus should be suspected of having scabies, but especially if a family member reports similar symptoms.3 A diagnosis can be made clinically if a burrow is detected at a typical predilection site and if the lesion itches severely. In this case, even a single burrow is pathognomonic.2 The diagnosis is confirmed by light-microscopic identification of mites, larvae, ova, or scybala (fecal pellets) in skin scrapings.1

TREATMENT

Treatment includes a scabicidal agent, an antipruritic agent such as a sedating antihistamine, and an appropriate antimicrobial agent in cases of secondary infection. Permethrin (Acticin), a 5% synthetic pyrethroid cream, is an excellent scabicide and is the preferred treatment.1 All family members and close contacts must be evaluated and treated, even if they do not have symptoms.1

References
  1. Chosidow O. Clinical practices. Scabies. N Engl J Med 2006; 354:17181727.
  2. Heukelbach J, Feldmeier H. Scabies. Lancet 2006; 367:17671774.
  3. Hengge UR, Currie BJ, Jäger G, Lupi O, Schwartz RA. Scabies: a ubiquitous neglected skin disease. Lancet Infect Dis 2006; 6:769779.
References
  1. Chosidow O. Clinical practices. Scabies. N Engl J Med 2006; 354:17181727.
  2. Heukelbach J, Feldmeier H. Scabies. Lancet 2006; 367:17671774.
  3. Hengge UR, Currie BJ, Jäger G, Lupi O, Schwartz RA. Scabies: a ubiquitous neglected skin disease. Lancet Infect Dis 2006; 6:769779.
Issue
Cleveland Clinic Journal of Medicine - 75(7)
Issue
Cleveland Clinic Journal of Medicine - 75(7)
Page Number
474, 478
Page Number
474, 478
Publications
Publications
Topics
Article Type
Display Headline
Generalized pruritus after a beach vacation
Display Headline
Generalized pruritus after a beach vacation
Sections
Disallow All Ads
Alternative CME
Use ProPublica
Article PDF Media

Acute facial purpura in an 82-year-old woman with a respiratory tract infection

Article Type
Changed
Thu, 04/19/2018 - 08:58
Display Headline
Acute facial purpura in an 82-year-old woman with a respiratory tract infection

An 82-year-old woman presents with facial purpuric lesions that developed while she was hospitalized for an acute respiratory tract infection characterized by severe paroxysms of nonproductive cough and dyspnea. The lesions appeared suddenly and spontaneously and were not associated with trauma. The patient denies pruritus or pain and is otherwise well. The remainder of her physical examination is within normal limits. She has hypertension, diabetes, and hyperuricemia but has had no recent changes in her medications.

Q: What is the most likely diagnosis?

  • Amyloidosis
  • Idiopathic thrombocytopenic purpura
  • “Cough purpura”
  • Purpura fulminans
  • Actinic purpura

A: The correct answer is cough purpura, a benign and nonthrombocytopenic eruption that appears to be related to a sudden rise in the venous and capillary pressure in the head and neck caused by a rise in intrathoracic pressure during coughing.1–3 Physical examination shows erythematous, nonblanching macules, smaller than 1 cm, distributed on the face and neck.

Figure 1.

Cough purpura is one form of the “mask phenomenon,”4 which is an unusual purpura of the relatively loose tissues of the face and neck occurring after vigorous vomiting, the Valsalva maneuver, parturition, prolonged coughing, or any other exertion that raises intrathoracic or abdominal pressure.1–5 The onset is acute. A workup for a coagulation or platelet defect is usually not required. Facial localization is unusual in most other forms of purpura, so its presence in addition to coughing should suggest cough purpura.

Recognizing cough purpura can help avoid misdiagnoses such thrombocytopenic purpura or purpura fulminans that could lead to ordering unnecessary tests, frightening the patient, and unnecessary confusion. The purpura fades spontaneously within 24 to 72 hours, and no treatment is needed.

References
  1. Kravitz P. The clinical picture of “cough purpura,” benign and non-thrombocytopenic eruption. Va Med 1979; 106:373374.
  2. Pierson JC, Suh PS. Powerlifter’s purpura: a Valsalva-associated phenomenon. Cutis 2002; 70:9394.
  3. Santiago Sánchez-Mateos JL, Aldanondo Fernández de la Mora I, Harto Castaño A, Jaén Olasolo P. Facial-cervical purpura. Rev Clin Esp 2007; 207:530532.
  4. Alcalay J, Ingber A, Sandbank M. Mask phenomenon: postemesis facial purpura. Cutis 1986; 38:28.
  5. Wilkin JK. Benign parturient purpura. JAMA 1978; 239:930.
Article PDF
Author and Disclosure Information

Sergio Vañó-Galván, MD
Department of Dermatology, Ramón y Cajal Hospital, University of Alcalá, Madrid, Spain

Ernesto Muñoz-Zato, MD
Department of Dermatology, Ramón y Cajal Hospital, University of Alcalá, Madrid, Spain

Address: Sergio Vañó-Galván, MD, Carretera de Colmenar km 9.100, 28034 Madrid, Spain; e-mail sergiovano@yahoo.es

Issue
Cleveland Clinic Journal of Medicine - 75(5)
Publications
Topics
Page Number
352
Sections
Author and Disclosure Information

Sergio Vañó-Galván, MD
Department of Dermatology, Ramón y Cajal Hospital, University of Alcalá, Madrid, Spain

Ernesto Muñoz-Zato, MD
Department of Dermatology, Ramón y Cajal Hospital, University of Alcalá, Madrid, Spain

Address: Sergio Vañó-Galván, MD, Carretera de Colmenar km 9.100, 28034 Madrid, Spain; e-mail sergiovano@yahoo.es

Author and Disclosure Information

Sergio Vañó-Galván, MD
Department of Dermatology, Ramón y Cajal Hospital, University of Alcalá, Madrid, Spain

Ernesto Muñoz-Zato, MD
Department of Dermatology, Ramón y Cajal Hospital, University of Alcalá, Madrid, Spain

Address: Sergio Vañó-Galván, MD, Carretera de Colmenar km 9.100, 28034 Madrid, Spain; e-mail sergiovano@yahoo.es

Article PDF
Article PDF

An 82-year-old woman presents with facial purpuric lesions that developed while she was hospitalized for an acute respiratory tract infection characterized by severe paroxysms of nonproductive cough and dyspnea. The lesions appeared suddenly and spontaneously and were not associated with trauma. The patient denies pruritus or pain and is otherwise well. The remainder of her physical examination is within normal limits. She has hypertension, diabetes, and hyperuricemia but has had no recent changes in her medications.

Q: What is the most likely diagnosis?

  • Amyloidosis
  • Idiopathic thrombocytopenic purpura
  • “Cough purpura”
  • Purpura fulminans
  • Actinic purpura

A: The correct answer is cough purpura, a benign and nonthrombocytopenic eruption that appears to be related to a sudden rise in the venous and capillary pressure in the head and neck caused by a rise in intrathoracic pressure during coughing.1–3 Physical examination shows erythematous, nonblanching macules, smaller than 1 cm, distributed on the face and neck.

Figure 1.

Cough purpura is one form of the “mask phenomenon,”4 which is an unusual purpura of the relatively loose tissues of the face and neck occurring after vigorous vomiting, the Valsalva maneuver, parturition, prolonged coughing, or any other exertion that raises intrathoracic or abdominal pressure.1–5 The onset is acute. A workup for a coagulation or platelet defect is usually not required. Facial localization is unusual in most other forms of purpura, so its presence in addition to coughing should suggest cough purpura.

Recognizing cough purpura can help avoid misdiagnoses such thrombocytopenic purpura or purpura fulminans that could lead to ordering unnecessary tests, frightening the patient, and unnecessary confusion. The purpura fades spontaneously within 24 to 72 hours, and no treatment is needed.

An 82-year-old woman presents with facial purpuric lesions that developed while she was hospitalized for an acute respiratory tract infection characterized by severe paroxysms of nonproductive cough and dyspnea. The lesions appeared suddenly and spontaneously and were not associated with trauma. The patient denies pruritus or pain and is otherwise well. The remainder of her physical examination is within normal limits. She has hypertension, diabetes, and hyperuricemia but has had no recent changes in her medications.

Q: What is the most likely diagnosis?

  • Amyloidosis
  • Idiopathic thrombocytopenic purpura
  • “Cough purpura”
  • Purpura fulminans
  • Actinic purpura

A: The correct answer is cough purpura, a benign and nonthrombocytopenic eruption that appears to be related to a sudden rise in the venous and capillary pressure in the head and neck caused by a rise in intrathoracic pressure during coughing.1–3 Physical examination shows erythematous, nonblanching macules, smaller than 1 cm, distributed on the face and neck.

Figure 1.

Cough purpura is one form of the “mask phenomenon,”4 which is an unusual purpura of the relatively loose tissues of the face and neck occurring after vigorous vomiting, the Valsalva maneuver, parturition, prolonged coughing, or any other exertion that raises intrathoracic or abdominal pressure.1–5 The onset is acute. A workup for a coagulation or platelet defect is usually not required. Facial localization is unusual in most other forms of purpura, so its presence in addition to coughing should suggest cough purpura.

Recognizing cough purpura can help avoid misdiagnoses such thrombocytopenic purpura or purpura fulminans that could lead to ordering unnecessary tests, frightening the patient, and unnecessary confusion. The purpura fades spontaneously within 24 to 72 hours, and no treatment is needed.

References
  1. Kravitz P. The clinical picture of “cough purpura,” benign and non-thrombocytopenic eruption. Va Med 1979; 106:373374.
  2. Pierson JC, Suh PS. Powerlifter’s purpura: a Valsalva-associated phenomenon. Cutis 2002; 70:9394.
  3. Santiago Sánchez-Mateos JL, Aldanondo Fernández de la Mora I, Harto Castaño A, Jaén Olasolo P. Facial-cervical purpura. Rev Clin Esp 2007; 207:530532.
  4. Alcalay J, Ingber A, Sandbank M. Mask phenomenon: postemesis facial purpura. Cutis 1986; 38:28.
  5. Wilkin JK. Benign parturient purpura. JAMA 1978; 239:930.
References
  1. Kravitz P. The clinical picture of “cough purpura,” benign and non-thrombocytopenic eruption. Va Med 1979; 106:373374.
  2. Pierson JC, Suh PS. Powerlifter’s purpura: a Valsalva-associated phenomenon. Cutis 2002; 70:9394.
  3. Santiago Sánchez-Mateos JL, Aldanondo Fernández de la Mora I, Harto Castaño A, Jaén Olasolo P. Facial-cervical purpura. Rev Clin Esp 2007; 207:530532.
  4. Alcalay J, Ingber A, Sandbank M. Mask phenomenon: postemesis facial purpura. Cutis 1986; 38:28.
  5. Wilkin JK. Benign parturient purpura. JAMA 1978; 239:930.
Issue
Cleveland Clinic Journal of Medicine - 75(5)
Issue
Cleveland Clinic Journal of Medicine - 75(5)
Page Number
352
Page Number
352
Publications
Publications
Topics
Article Type
Display Headline
Acute facial purpura in an 82-year-old woman with a respiratory tract infection
Display Headline
Acute facial purpura in an 82-year-old woman with a respiratory tract infection
Sections
Disallow All Ads
Alternative CME
Use ProPublica
Article PDF Media